Saturday, July 12, 2014

Clarke on the stock caricature of First Cause arguments


W. Norris Clarke’s article “A Curious Blind Spot in the Anglo-American Tradition of Antitheistic Argument” first appeared in The Monist in 1970.  It was reprinted in his anthology The Creative Retrieval of St. Thomas Aquinas: Essays in Thomistic Philosophy, New and Old, which was published posthumously in 2009.  I only just read the essay, and I did so with embarrassment and gratification.  Embarrassment because I found that something I’ve been harping on for a few years now had already been said by Fr. Clarke over 40 years ago.  Gratification because I found that something I’ve been harping on for a few years now had already been said by Fr. Clarke over 40 years ago.

The stock caricature in question is, of course, the “Everything has a cause, so the universe has a cause” argument.  As I’ve pointed out many times (e.g. here and here), no major proponent of the idea of a First Cause ever actually defended this stupid argument.  Indeed, all the major proponents of arguments for a First Cause would reject the claim that “everything has a cause,” and on entirely principled rather than ad hoc grounds.  Hence the stock retort to this caricature has no force whatsoever against their actual arguments.  That stock retort is of course to ask “If everything has a cause, then what caused God?” and then to suggest that if God need not have a cause, then neither need the universe have a cause.  Maybe, those who attack this caricature suggest, it is the universe itself (or the event that gave rise to it) that is the first or uncaused cause.

The “curious blind spot” Clarke is referring to is contemporary Anglo-American philosophers’ amazing inability or unwillingness to see that in routinely trotting out this objection they are attacking a straw man that bears no interesting relationship whatsoever to what writers like Aristotle, Aquinas, Leibniz, et al. actually said. 

In previous posts, I’ve given many examples of philosophers who attack the straw man First Cause argument.  They include Bertrand Russell, Steven Hales, Nigel Warburton, and (as I showed in a post discussing several examples at once) Daniel Dennett, Robin Le Poidevin, Graham Priest, Michael Martin, Simon Blackburn, Jenny Teichman and Katherine Evans.  Clarke offers several further examples from philosophy textbooks of the mid twentieth century, including John Hospers’ widely used An Introduction to Philosophical Analysis.  As Clarke indicates, Russell’s Why I Am Not a Christian may be the source from which many subsequent writers learned this caricature and the stock reply to it.  Clarke also notes that Russell in turn seems to have gotten the idea from John Stuart Mill, who in turn got it from his father James Mill.  Clarke thinks that David Hume, who in the Dialogues Concerning Natural Religion attacks something like the stock straw man First Cause argument, may be the first well-known writer to do so.  Clarke writes:

Let it first be agreed without qualification that if one does admit the principle “Every being has a cause,” then the refutation is inescapable and devastating.  But the very ease of this refutation, if nothing else, should have aroused some suspicions in the minds of its users, one would have thought, as to whether their supposed opponents were actually using this principle.  And it is in itself a highly suspicious fact that no one among the many in this Hume-Russell tradition whom I have read ever quotes any specific theistic philosopher who does make use of it.  So constant is this pattern, in fact, that I am willing to wager that this family trait is found also in those I have not yet run across. (p. 55, emphasis added)

As I have noted in the earlier posts cited, the pattern in question certainly has continued in the 40 plus years since Clarke wrote.  Critics regularly attack the straw man without citing anyone who has ever defended it.  (Le Poidevin even admits that no one has actually defended it!)  After falsely accusing proponents of the First Cause argument of contradicting themselves by denying that God has a cause, Hospers smugly writes:

Many people do not at once see this because they use the argument to get to God, and then, having arrived at where they want to go, they forget all about the argument… (quoted by Clarke at p. 52)

But who exactly are these “many people”?  The critics do not tell us.  It’s tempting to conclude (paraphrasing Hospers) that these critics do not see that no one has ever really defended the straw man they attack because, having arrived at where they want to go -- a way of dismissing Aristotle, Aquinas, Leibniz, et al. tout court and thereby avoiding commitment to a divine First Cause -- they forget all about what these writers actually said.  Says Clarke:

We can only conclude, then, that the Hume-Russell tradition of anti-theistic argument, on this point at least, somehow got off to a bad start by completely misunderstanding and misrepresenting the very argument it was trying to refute, and that it has continued to repeat itself ever since, talking only to itself, and without ever bothering to inquire whether the supposed other party to the debate was still there at all, or had ever been there.  In a word, it has become a tradition in the worse sense of the word, truly in a rut and apparently unaware of it. (p. 59)

Confirming evidence of this is provided by Steven Hales’ response to my recent criticism of him for peddling the straw man.  Prof. Hales wrote:

I do find it surprising that Professor Feser chooses to hang his hat on the Cosmological Argument of all things, an argument that the vast majority of contemporary philosophers consider risible, but I suppose that no interesting philosophical argument is ever truly dead.

But of course, the reason “the vast majority of contemporary philosophers consider [the argument] risible” is precisely because what they know of it is the stupid straw man version peddled in books like Hales’ rather than what proponents of the cosmological argument have actually said!  It’s a vicious circle.  “We know the cosmological argument in general is too stupid to be worth taking seriously because the version we learned from the textbooks is so easily refuted; and we know there aren’t any other versions worth looking into, because the cosmological argument in general is too stupid to be worth taking seriously.”  This tells you nothing about the value of the cosmological argument, and everything about the value of the conventional wisdom in academic philosophy.

In fact, as Clarke notes, Aquinas explicitly denies that everything has a cause.  He held that “to be caused by another does not appertain to a being inasmuch as it is being; otherwise, every being would be caused by another, so that we should have to proceed to infinity in causes -- an impossibility…” (Summa Contra Gentiles II.52.5).  For writers like Aristotle, Plotinus, and Aquinas and other Scholastics, it is not the fact of something’s existence as such, or of its being a thing per se, that raises causal questions about it.  It is only some limitation in a thing’s intrinsic intelligibility that does so -- for example, the fact that it has potentials that need actualization, or that it is composed of parts which need to be combined, or that it merely participates in some feature, or that it is contingent in some respect.  Hence these writers would never say that “everything has a cause.”  What they would say is that every actualization of a potential has a cause, or whatever is composite has a cause, or whatever has a feature only by participation has a cause, or whatever is contingent has a cause.  

Accordingly, when they arrive at God via a First Cause argument, there is no inconsistency, no sudden abandonment of the very premise that got the argument going.  Rather, the argument is that the only way to terminate a regress of actualizers of potentials is by reference to something which is pure actuality, devoid of potentiality, and thus without anything that needs to be or even could be actualized; or it is that a regress of causes of composed things can be terminated only by something which is absolutely simple or non-composite, and thus without any parts whose combination needs to be or indeed could have been caused by anything; or that the only way to terminate a regress of things that cause other things to participate in being is by reference to that which just is being itself rather than something which merely has or participates in being, and thus something which neither needs nor could have had a cause of its own being; or that the only way to terminate a regress of causes of contingent things is by reference to something absolutely necessary, which by virtue of its absolute necessity need not have and could not have had something impart existence to it; and so forth.

Whatever one thinks of these sorts of arguments, there is no inconsistency in them, nor any ad hoc exceptions to general principles.  The only way to accuse them of either fault is by reading into them the silly straw man argument that their proponents would reject.

How did the Hume-Russell straw man tradition ever get started in the first place?  I noted in another recent post that Descartes’ “preservation” argument, an eccentric and now little-known variation on the cosmological argument, implies that there is a sense in which everything has a cause -- though it does not explicitly appeal to that claim as a premise, and it does not make an exception in the case of God since it regards Him as self-caused.  Clarke discusses this argument in some detail and shows that while Descartes’ development and defense of the argument in the Replies is complicated and confusing, at the end of the day even he does not appear to be saying quite the sort of thing that the Hume-Russell straw man attributes to First Cause arguments.  What Descartes is saying is something closer to a version of the principle of sufficient reason (PSR), on which everything has an explanation.  And in the case of cosmological arguments that appeal to PSR (like Leibniz’s), the Hume-Russell style objection cannot get off the ground, because these arguments do not and need not make any exception in the case of God.  They hold that absolutely everything has an explanation.  In the case of contingent things, the explanation lies outside the thing, and in the case of a necessary being, the explanation lies in the thing’s own nature.  Again, whatever one thinks of such arguments, there is no inconsistency in them, nor any ad hoc exception to a general principle.

Clarke suggests that Descartes blurred the distinction between a cause and a sufficient reason, and that Spinoza (who also thought of God as self-caused) did the same.  What they really meant was something like “Everything has an explanation,” where they make no exception in the case of God.  But since they use the language of “cause,” it sounds like they are saying that “Everything has a cause” in the usual sense of an efficient cause which is distinct from its effect.  And of course that is the sort of cause that God is traditionally said not to have, and which Descartes and Spinoza themselves would deny that he has (even if they think he does have a “cause” in the sense of a sufficient reason).

Clarke suggests that what Hume did was essentially to confuse these two senses of “cause,” taking the rationalist claim that “everything has a ‘cause’-in-the-sense-of-a-sufficient-reason” to be identical to the claim that “everything has a ‘cause’-in-the-sense-of-an-efficient-cause-distinct-from-itself. “  In fact no defender of the cosmological argument ever made the latter claim, but since Descartes and Spinoza made the former claim it seemed to Hume as if someone had made it.  He then essentially made the further step of attributing this thesis to proponents of the cosmological argument in general.  And then, since proponents of the cosmological argument in general do deny that God has a ‘cause’-in-the-sense-of-an-efficient-cause-distinct-from-himself, the claim that proponents of the argument were contradicting themselves seemed to have force.  But as Clarke says:

Thus the First Cause argument for the existence of God which the Hume-Russell tradition so devastatingly attacks is indeed an inviable metaphysical monster.  But it is a monster of their own fabrication, not that of any reputable theistic philosopher.  It is actually a kind of hybrid of both the traditional Scholastic and Cartesian rationalist traditions, which would make sense in neither and be repudiated by both.  (p. 62)

Clarke goes on to note that while Hume may have had some excuse for this error given the confusing nature of Descartes’ terminology, “it is much harder to excuse his successors in this tradition, with all the resources of historical scholarship and linguistic analysis at their disposal, for perpetuating this confusion” (p. 62).  And again, Clarke wrote this over 40 years ago.  In the decades since, lip service to and indeed genuine knowledge of the history of philosophy has dramatically increased within Anglo-American analytic philosophy, and still this absurd caricature of the cosmological argument (not to mention many other equally stupid caricatures of traditional arguments for the existence of God, the immateriality and immortality of the soul, and natural law conclusions in ethics) are routinely and matter-of-factly peddled by academic philosophers.  By people who will, of course, assure you that intellectual dishonesty is all on the side of religious believers. 

And unfortunately, the Hume-Russell straw man has so deeply distorted general understanding of the cosmological argument that even some theists -- indeed, even some sympathizers with the cosmological argument -- feel they have to treat it as if it had something to do with the arguments of Aristotle, Aquinas, Leibniz, et al.  Consider Alex Pruss’s article “The Leibnizian Cosmological Argument” in The Blackwell Companion to Natural Theology.  In general it is (as, of course, Alex’s work typically is) excellent.  But Alex says that “a typical cosmological argument faces four different problems,” one of which he describes as follows:

The third difficulty is the Taxicab Problem, coming from Schopenhauer’s quip that in the cosmological argument, the Principle of Sufficient Reason (PSR) is like a taxicab that once used is sent away.  The difficulty here is in answering what happens when the explanatory principle… gets applied to the First Cause.  A popular formulation is: “If God is the cause of the universe, what is the cause of God?”  Typical solutions argue that the case of the First Cause is different in some way that is not merely ad hoc from the cases to which the explanatory principle was applied. (pp. 24-25)

Alex goes on to argue that this “problem” can indeed be solved, but I think he should never have treated it as a “problem” for the argument in the first place.  Suppose critics of Darwinism routinely asserted that Darwinians claim that a monkey gave birth to the first human baby, and also routinely went on to ridicule this claim as evidence of the “risibility” of Darwinism.  Would it be a good idea for a defender of Darwinism to say that “a typical Darwinian argument faces four different problems, one of which is the Monkey Problem,” and then go on to offer a solution to this “Monkey Problem”?  Of course not, because the “Monkey Problem” is a complete fabrication that no version of Darwinism ever needed a “solution” to.  The proper response would be relentlessly to hammer this point home, not to dignify the objection by treating it as if it were something other than an attack on a straw man.  That only reinforces the misunderstanding in question in the very act of trying to resolve it.  But the same thing is true of the bogus “Taxicab Problem.”  (I think something similar could be said, for that matter, of the other three “problems” Alex refers to in the article.  They all concern issues that defenders of cosmological arguments are typically addressing head on from the start, not “problems” that remain to be solved after the arguments have been given.)

I’ll give Fr. Clarke the last word, by quoting a passage that I think conveys the correct attitude to take toward those who attack the Hume-Russell straw man.  I think a willingness to assent to what Clarke says here provides a useful test of the competence and intellectual honesty of any atheist and of any professional philosopher:

[W]e are here in the presence of a philosophical tradition that is truly in a self-repetitive rut, a tradition that has long since ceased to look outside of itself to check with reality and see whether the adversary it so triumphantly and effortlessly demolishes really exists at all… [I]t would seem to be high time that those who still follow this particular tradition of antitheistic argument should have the grace and humility to acknowledge that their argument is dead, and let us get on with more substantive problems with regard to philosophical argument for and against the existence of God.  (pp. 62-63)

203 comments:

  1. But it also doesn't show that God is omnipotent, omniscient, and omnibenevolent!

    Nailed it.

    ReplyDelete
  2. Yeah, Alex calls that the "Gap Problem," though he goes on right away to note that Aquinas fills this supposed "gap" immediately after giving the Five Ways with a series of "careful and elaborate arguments." And of course, other defenders of the cosmological argument do the same. In which case there is, of course, no "Gap Problem" in the first place. In which case, why are we talking about a "Gap Problem"?

    ReplyDelete
  3. I'm interested to know how this type of argument could continue to exist for more than an instant in modern philosophy since, aside from the almost cosmic degree of historical and shall we say… philosophical ignorance it displays, the notion of necessary beings, in these cases abstract objects, deriving their PSR from within so to speak has become common parlance with the advent of Possible World metaphysics.

    Speaking of criticisms of the Cosmological Argument is there any chance we will see that post in response to some of Graham Oppy's criticisms you mentioned working on whilst answering J Lowder?

    ReplyDelete
  4. I think there is often a tendency among some atheist circles to attack the Pastor Bob argument (i.e. the everything has a cause strawman, with additional "difficulties" like the gap problem) and somehow take that argument to basically be the Cosmological Argument (Didn't Le Poidevin pretty much do exactly this?). After all, if you corner your average Christian w/ regards to the existence of God, they might appeal to the Pastor Bob CA, and when you have enough Christians doing this en masse, it might establish an unfortunate pattern. Of course, that doesn't excuse the academic philosophers who ought to be aware of the more nuanced arguments, but it might help explain the phenomena seen in atheism at large. Once that is established, the groupthink becomes hard to get rid of.

    This is all entirely speculative, of course, but I think it is at least part of the reason you see the straw man crop up so much in dialogue.

    ReplyDelete
  5. I’m trying to streamline my response to the “What caused God?” argument.


    To ask what caused a first cause is obviously senseless. To ask about something other than a first cause is not to ask about God. The proof has instead to do with whether there is anything that is independently real, and with the nature of that thing.

    Still needs some work.

    ReplyDelete
  6. Leibniz' took a methodological approach to the PSR version which would have prevented this sort of objection even arises i.e. he gave the standard Modal Ontological Argument with the standard 'If Possible then Actual' conclusion and then sought to employ his Cosmological Argument from the PSR. Now I’m not saying one has to or even ought to do that, but that Schopenhauer, who never missed a chance to wheel out the useless Kantian criticism about ‘Existence not being a predicate’, could not have been ignorant of it.

    ReplyDelete
  7. Dr. Feser,

    I thought I understood the Cosmological Argument, but now I'm puzzled once again. You say that "everything has a cause" is not a premise of the argument, but again what about God? Is he not a cause of himself? Or is he uncased? I'll have to reread your Aquinas, but I'm pretty sure the Classical Theism holds that God is the cause of himself. And if so, then indeed everything seems to have a cause.

    ReplyDelete
  8. Excellent points here.

    I'll admit that I am much more sympathetic to the existence of a "gap problem," though, not because theistic philosophers haven't attended to it, but because apologetics tends to spend less time treating it than cosmological arguments themselves. For an apologist truly to put pressure on an atheist, the divine attributes require a very thorough treatment which they often are not given.

    Part of the issue, I think, is that something like Aquinas's First Way is actually not impossible to mobilize with rather modest metaphysical groundwork. (This is a point Oderberg makes in his article '"Whatever is Changing is Being Changed by Something Else": A Reappraisal of Premise One of the First Way'.) The same is not exactly true of deriving some of the divine attributes (particularly omniscience, but omnipotence requires fleshing out the principle of proportionate causality in a deeper way than the First Way alone requires, and goodness requires an understanding of the transcendentals, etc.). So apologists often do the easy part and leave the rest hanging.

    ReplyDelete
  9. @ Kirill Nielson

    Some of the most significant strands of classical theism characterize God as the Uncaused Cause. So yes, classical theism certainly does regard God as uncaused.

    As Feser points out, there may be some senses in which God could be said to be caused. Spinoza and Descartes both characterized him as self-caused. But that is a different sense of cause than used in the term "Uncaused Cause"; it is more akin to explanation than efficient cause.

    ReplyDelete
  10. I'll admit that I am much more sympathetic to the existence of a "gap problem," though, not because theistic philosophers haven't attended to it, but because apologetics tends to spend less time treating it than cosmological arguments themselves.

    I should clarify: the gap problem is clearly worthless in the form it is usually raised, ie. "But there's absolutely no reason to believe that that's the God of Christianity!" Since this is usually not accompanied by some argument that the uncaused cause lacks the attributes of the Christian God, the basis of such statements seems to be the inability of the objector to imagine what sort of arguments would establish them... but it's pretty rare that someone would have the audacity to claim that there is no basis for a claim because one can't imagine what the reason would be. (By "pretty rare" I mean it probably wouldn't happen unless the argument in question were an argument for theism.)

    ReplyDelete
  11. Greg,

    Thank you. What is confusing me is distinction between causes and explanations. I quite often see the phrase, "God himself is the explanation of his own existence." You know, because everything that exists needs an explanation.

    I have a strong tendency to treat "reason" and "explanation" as the same thing.

    ReplyDelete
  12. Continued.

    By the way, it gets even trickier when you read Wiki on the Principle of Sufficient Reason.

    http://en.wikipedia.org/wiki/Principle_of_sufficient_reason

    See, it outright says that everything that exists needs a cause. Not an "explanation, a "cause."

    I'm aware that Wiki is an embarrassing source to cite, and that there are different versions the PSF. So, which formulation do you think is the most accurate and best lines up with the Cosmological Argument?

    ReplyDelete
  13. @Kirill Nielson:

    Greg is of course quite right that "cause" is sometimes taken broadly to include "explanation," but he's also right to note that this is different sense than "efficient cause." The latter is what's at issue in Aquinas's arguments, and in this sense he certainly holds that God is not "self-caused." That anything should be its own efficient cause, or actualize its own potencies, he regards as simply absurd.

    ReplyDelete
  14. Aquinas fills this supposed "gap" immediately after giving the Five Ways with a series of "careful and elaborate arguments."

    I wish he had ended the argument with "...and this all men call God. Details to follow."

    ReplyDelete
  15. PART 1:
    Dear Doctor Feser,
    Would you please quote or give the exact reference to Hume’s rejection of the “first cause” argument and specifically what it was he rejected? You are probably right that he got confused with Descartes’ “explanation” which I also find difficulty in really differentiating from “cause”. Aquinas uses the ‘necessity of cause’ simply to show that everything a human being can know is fundamentally in some sense an accident whereas human beings find it extremely difficult to understand how absolutely everything and anything is contingent without some kind of “necessary” principle to give substance to it, i.e., to anchor it in some undeniable “First Principle”. Yet any “First Principle” can be denied wherein the opponent calls for the contest of experiment showing whether the First Principle works or does not work. First, that is ALL the experiment does. Second, the fundamental principle of an experiment is that it is fundamentally repeated to show whether it STILL results in the same conclusion. Third, the second is practically necessary because any experiment only covers a ‘relevant’ selection of matters included and points to be made, that is, there is always the possibility that something ‘new’ may suddenly show up. And mere “practicality” is NOT itself “fundamental truth” no matter how inconvenient that objection is. Aristotle in POSTERIOR ANALYTICS, Bk 2, chaps 18-20, “pictured” the establishment of a “First Principle” as the reaction to a rout in battle where the disciplined soldiers turn and make a stand making the other fleeing soldiers start to do the same until they can make a proper defense. “First Principles” are a matter of decisive commitment and invention, created out of other words already in use, and tested by material experience. But they are never infallible or “undeniably true” because, if for no other reason, they can in fact be ‘denied’.¶
    -x-
    The reason I ask for a specific reference on Hume is that, one, while he was writing A TREATISE ON HUMAN NATURE in France under the sponsorship of the Chevalier Andrew Michel Ramsey, a Scot Presbyterian convert to Catholicism (THE LIFE OF DAVID HUME by E. C. Mossner , chap. 8, “Tranquillity in France”) who, though absolute philosophical opposites, got along very well personally, as well as many prominent French scholars and the staff of Descartes school , the Jesuit Collège Royal Henry-Le-Grand at La Flèche. Ergo, I assume he had to have a fairly thorough direct knowledge of Scholasticism Two, Diderot wrote about “M. Hume at the table of Baron d’Holbach”, a dinner for atheists, (chap. 33) “I don’t know for what purpose the English philosopher took it into his head to remark to the Baron that he did not believe in atheists, that he had never seen any…” This goes with another anecdote I cannot find now where, before the dinner, d’Holbach and Diderot asked Hume if he were an atheist, that it was alright to say so since they were all atheists. But Hume insisted that, no, he “was a philosophical theist”. Ergo it would seem that if Hume rejected the “first cause” argument’ it had nothing to do with rejecting “God” as a First Principle of some sort. Ergo he should not be associated with Russell. ¶

    ReplyDelete
  16. PART 2
    Now Russell could be rash in his judgments of other philosophers like Aquinas and Heidegger (I know he later recanted on Heidegger more or less saying he did not understand him), but, prompted by his intellectual conflict with Wittgenstein where Wittgenstein always had the upper hand (and Russell admitted it) he constantly revised his PRINCIPIA MATHEMATICA TO ACCOUNT FOR Wittgenstein’s and others objections until in conflict with Henri Poincaré on the “Cretan liar paradox” he came up with the interesting idea of the “propositional function” which supposedly solved that problem that had already been “solved” by the Neo-Platonists starting with Porphyry’s ISOGOGE where Porphyry stated a genus could not be a species to itself which reflects Aquinas statement every CONTINGENT thing must have a cause and supported Porphyry in his commentary on the “Liber de Causis” which was mainly quotes from Proclus’ ELEMENTS OF THEOLOGY where Proclus [and Plotinus] used Porphyry’s statement to justify NOT being able to define the “One” in any way whatsoever, though everything else is “caused” by it, because explain the “One” would in effect 1] make it a “genus” when no concepts at all can apply to it, and 2] the empty logical ‘slot’ the “One” occupies in the emanation of being cannot be subordinate to itself just as Porphyry said of a genus not being its own species. And, though Russell is probably turning in his grave now, this SEEMS to me to be similar to Russell’s “propositional function” which must exist BEFORE you can propose “x is y” or whatever (BERTRAND RUSSELL:THE SPIRIT OF SOLITUDE by Ray Monk, pp. 185-88).

    ReplyDelete
  17. Dear Doctor Feser,
    While I may temporarily have your attention, what is a good, solid article on what EXACTLY "revealed theology" is (page 2 your AQUINAS).
    Truly,
    Gary C. Moore

    ReplyDelete
  18. Another great post, Doc.

    ReplyDelete
  19. Question: Could you list a few *substantial* introductory philosophy books that are really good (within, say, the last 50 years). I would like a book that treats the main branches in philosophy in a more than cursory way found in most "introductory books". Maybe something like the Hospers one. Probably requiring the book to be 600+ pages. No anthologies, please, or edited collections.

    ReplyDelete
  20. I e-mailed Hales about this, in a vain effort to get him to publicly acknowledge his mistake. His response was essentially "Oh, you're a Feser devotee defending the First Way. Good luck with that." He then went on to quote Aquinas on the SECOND Way, from an order of efficient causes, in an attempt to prove that he did in fact hold the "everything has a cause" premise.

    If ever anything deserved to be called risible...

    ReplyDelete
  21. @ Kirill

    See, it outright says that everything that exists needs a cause. Not an "explanation, a "cause."

    I'm aware that Wiki is an embarrassing source to cite, and that there are different versions the PSF. So, which formulation do you think is the most accurate and best lines up with the Cosmological Argument?


    As I said, there are senses of "cause" in terms of which the principle of sufficient reason can be formulated. I think that this obscures a distinction between efficient causes and explanations, which are different, since the PSR cannot be formulated in terms of efficient causes, but that is generally what people think of when they read "cause" without qualification.

    ReplyDelete
  22. @ Curio

    If ever anything deserved to be called risible...

    Hey... if you decide before the fact that you'll laugh no matter what anyone says, then anything could be risible.

    ReplyDelete
  23. He then went on to quote Aquinas on the SECOND Way, from an order of efficient causes, in an attempt to prove that he did in fact hold the "everything has a cause" premise.

    Well, at least he's given up the unprofessional defense that he gave in the comments threads here, that it was just for an introductory text. It's pretty typical, though -- the Second Way itself repeatedly refers to first efficient cause, ultimate efficient cause in a regress, and so forth, so it doesn't presuppose anything like it, either -- for these mistakes to get covered with some attempt at three-card monte.

    ReplyDelete
  24. @Curio:

    "He then went on to quote Aquinas on the SECOND Way, from an order of efficient causes, in an attempt to prove that he did in fact hold the 'everything has a cause' premise."

    That's quite a job, since (in the link Ed has already provided) Aquinas expressly states otherwise: "Therefore, that being which is subsisting must be uncaused."

    ReplyDelete
  25. In one sense, Christian philosophers have an argument for these situations which they actually cannot use, even if they believe it (as I do). Those who will to know of God will be taught by Him, according to Christ. Therefore, some cognitive problems could actually be volitive problems--a sort of moral unwillingness, a deliberate blindness if an argument leads one where one does not wish to go. (Ed alluded to this in the post.) We can fall prey rather easily to this, given innate tendencies toward sin.

    Why cannot we use this argument? Well, it seems we are accusing the skeptic of bad faith: something like, "You know you really COULD understand this argument, or at least not mischaracterize it, if you weren't simply refusing to do so." Even if this were true (as I said, I often think it is), isn't it too judgmental, not to mention a cop-out from the argument itself?

    ReplyDelete
  26. Fr. Clarke:

    The proposition "No being can cause itself" is in no way reducible to "Every being has a cause," since implicit in the former is the qualifying proviso "No being which needs a cause can be the cause of itself." This obviously asserts nothing as to whether every being requires a cause or not, only that every being that does require a cause must be caused by another. It is as though one were to maintain that "No man can be married to himself" logically implies "Every man is married" -- an obvious non sequitur. -- p. 54 (Fr. Clarke's emphases)

    (The article can be read in its entirety here (after scrolling down to p. 48).)

    ReplyDelete
  27. picked up on the internet:
    The Doctrine of Revelation

    While treated here as the third, among Protestants, this doctrine has traditionally been the first. Both John Calvin and Karl Barth began their theologies with the question of how one comes into knowledge of God. The (Presbyterian) Westminster Confession of Faith (1646) and the Second [Baptist] London Confession of 1677 began with Scripture. It is an important doctrine because Jn. 17:3 says that it through knowing God and Jesus Christ that one obtains eternal life. Yet there is a difference between knowing God and knowing about God. The passage in John deals with the former--knowing in an experiential, relational way. That was the understanding of "knowing" that characterized the Hebrew thinking peoples from whom our Scriptures came. One must know God like one knows one's parents or children--not the way that one might know the answer to a question on a television game show.
    Are the Bible and the Word of God really the same? Most evangelicals today will usually answer "Yes" to this question, but one must ask whether such an answer is really biblical. The Ethiopian Eunuch in Acts 8 was struggling to understand Isa. 53 and was making no headway. He was reading the Scripture, but he was not getting the Word of God. Along came Philip, who was sensitive to what God was doing and who "preached" to him. Suddenly the Eunuch understood the Word of God with clarity and was baptized on confession of faith. Put another way, the Eunuch actually come to know God because he had encountered the transforming power of the Word of God. This example tells us that the "Word of God" was not the Isaiah text itself. Yes, the Scripture was the center of the narrative, but the presence and work of the evangelist (Philip) and the Holy Spirit (who appears repeatedly in this passage) were also necessary for the Eunuch to appropriate the Word of God in this case.
    The Doctrine of Revelation must embrace the whole process of God revealing himself to us. The role of the Scriptures is foundational, but they are not the whole of the process. It is, of course, possible for God to reveal himself completely outside of the Bible, as he did to Abraham. But without having the Scriptures to validate one's experience with God one can never be sure that one is appropriating the knowledge of God or of the pizza one may have had the night before. So the Bible has a part, but to read the Bible without the other ingredients is to read mere human words, devoid of any possibility of achieving the Knowledge of God. Calvin summarized it this way--one needed the "spectacles of faith" and the Holy Spirit to read the Bible properly.
    -x-
    GCM: I thought Karl Barth believed "God" was "wholly other" which would make him utterly beyond the purview of human language?

    ReplyDelete
  28. How did the Hume-Russell straw man tradition ever get started in the first place?

    Because everyone has been thoroughly convinced by Scholastic philosophy, of course! Even people who don't know it, which is apparently an awful lot of them. If one begins from "everything has a cause except for either God (or maybe gods) or the universe", it immediately follows that theism is true — either monotheism (or perhaps polytheism) or pantheism. Yet nobody nowadays ever considers pantheism to be theism (even though it's right there in the name, people!!). And this is because the Scholastic heritage in which even decadent moderns are still steeped to a very large amount utterly put paid to the idea that the "Universe" came anywhere to close to qualifying as the Uncaused Cause. Everybody just knows that the universe is Not God... so positing it as an alternative to monotheism sounds like something not theistic at all.

    The OFloinn: I wish he had ended the argument with "...and this all men call God. Details to follow."

    Or to come at it the other way around, everyone has been thoroughly convinced of the attributes of God, so much so that when faced with a vague Prime Mover, those who do not know the reasoning nevertheless are confident that there's more to being God. 'Tis a pity that so many people are in so much agreement with the conclusions that they reject the arguments, but such is life in the Information Age.

    ReplyDelete
  29. I wish atheists were less boring. Their arguments against religion always boil down to strawmen, which they serve with a side of hysterical ranting about "Reason". If only they were more rebellious and interesting.

    ReplyDelete
  30. A good experiment sometime would be to produce an "anti-theist argument" that just used the five ways to establish an unmoved mover that couldn't be created, was responsible for laws and motion and, etc. The only change you'd make is call it 'The universe' rather than 'God' at the end, and argue that this makes God completely superfluous. Throw in some nasty anti-theist barbs, etc.

    I wonder how many would go for it.

    ReplyDelete
  31. It would make the front page of the NYT, Crude. Followed by, Ha!

    ReplyDelete
  32. @Kirill Neilson

    "By the way, it gets even trickier when you read Wiki on the Principle of Sufficient Reason.... it outright says that everything that exists needs a cause. Not an "explanation, a "cause."
    "


    The Principle of Sufficient Reason is not even properly formulated to include causes. The Principle of Causality, however, is (every effect has a cause).

    The Principle of Sufficient Reason states that every being has within itself or in another the sufficient reason for its being (or existing).

    The Principle of Causality can be seen rather to follow from PSR (because effects never have within themselves exclusively the sufficient reason for their being or existing but always require some other for their being; that is, their cause).

    ReplyDelete
  33. @Crude:

    A good experiment sometime would be to produce an "anti-theist argument" that just used the five ways to establish an unmoved mover that couldn't be created, was responsible for laws and motion and, etc. The only change you'd make is call it 'The universe' rather than 'God' at the end, and argue that this makes God completely superfluous. Throw in some nasty anti-theist barbs, etc.

    The universe changes, though, at least in part. Better to replace 'the universe' with 'the laws of physics'. The self-existent, unchanging laws that bring forth the universe into being from nothing, actualize potentialities, and necessitate everything that happens.

    This it would seem is what many atheists view these supposed laws as in the first place: a sort of prime mover, devoid of love, mercy, personhood, intellect, offering no redemption, salvation, grace, being, in effect, that which all men call God, without being God.

    ReplyDelete
  34. @Logo

    So their conception of the universe basically boils down to a dead beat dad Creator, though of course concealed in scientific sounding terms to make it seem emotionally or personally neutral?

    That's interesting and oddly human. One wonders whether or not it's more of a kind of self-fulfilling prophecy than an objective and discovered truth of science in that case.

    ReplyDelete
  35. This it would seem is what many atheists view these supposed laws as in the first place: a sort of prime mover, devoid of love, mercy, personhood, intellect, offering no redemption, salvation, grace, being, in effect, that which all men call God, without being God.

    You mean Spinoza's view of God.

    ReplyDelete
  36. @ Mr. Green

    "Or to come at it the other way around, everyone has been thoroughly convinced of the attributes of God, so much so that when faced with a vague Prime Mover, those who do not know the reasoning nevertheless are confident that there's more to being God. 'Tis a pity that so many people are in so much agreement with the conclusions that they reject the arguments, but such is life in the Information Age."

    I've long felt that something similar has happened to our culture that prevents much evangelizing from being effective. We, as a society, are too well immersed in holy water; people don’t feel the need for redemption because they usually were baptized as infants and have had it too good, so to speak. It’s not really a bad thing that this has happened, but it makes it hard to convince people that they are heathens when they really are heretics. It’s definitely a unique situation.


    @ Crude

    “A good experiment sometime would be to produce an "anti-theist argument" that just used the five ways to establish an unmoved mover that couldn't be created, was responsible for laws and motion and, etc. The only change you'd make is call it 'The universe' rather than 'God' at the end, and argue that this makes God completely superfluous. Throw in some nasty anti-theist barbs, etc.

    I wonder how many would go for it.”

    Arguably that’s what 99% of atheists/agnostics have done in the past; we’re just in an interesting phase because the atheists haven’t realized that they can’t use sola scientia any more consistently than protestants can use sola scriptura. Eventually they will have to return to at least a minimal metaphysic, and that’s when they’ll start having to use those arguments to avoid looking silly.

    ReplyDelete
  37. I specifically remember Steven Hales leaving a comment on a blog where he claimed that the persistence of Theism demands special psychological explanation rather than serious philosophical engagement. I believe it was in response to the media blitzkrieg following Keith Parsons' initial "I'm quitting philosophy of religion" charade.

    Now if there is ever a candidate for some serious retortion, this is it.

    His arguments against Theism are strictly undergraduate. So, with respect to Hales, we need a special psychological explanation rather than serious philosophical engagement.

    So let's entertain a more basic question: Why can people like Steven Hales and Alex Rosenberg and Daniel Dennett continue to get away with such unbelievably bad and ill-informed arguments, and in a discipline that allegedly puts a premium upon "rigor"? If I were a social psychologist this would be a pay-day.

    Putative psychological explanation: There is no relevant peer-directed social pressure placed upon naturalists or anti-theists to change their minds. Sothey know they can just phone it in and nobody will bat an eye, even if their arguments completely stink. For purely social reasons, the bar they have to clear is much lower than the bar raised for anti-naturalits and Theists.

    ReplyDelete
  38. Logo,

    The universe changes, though, at least in part.

    Yeah, that's true. I'm sure a good alternative can be found, like some platonized 'laws'. Without saying they're platonized.

    This it would seem is what many atheists view these supposed laws as in the first place: a sort of prime mover, devoid of love, mercy, personhood, intellect, offering no redemption, salvation, grace, being, in effect, that which all men call God, without being God.

    Well, the twist here would be that the five ways would establish some of those things without them realizing it. I'll give it a shot in conversation sometime, see who's caught.

    ReplyDelete
  39. Primum objectum est ens ut commune omnibus - John Duns Scotus

    ReplyDelete
  40. Speaking of retorsion, atheists often accuse believers of motivated reasoning. The belief, for example, in an afterlife is said to be motivated by comfort seeking. But which of the following ideas of death is the most comforting;

    1) Death as dreamless sleep - the version in which the atheist believes.
    2) Death with the possibility of hell.
    3) Reincarnation, with the troubles of this life repeated indefinitely.

    I find the notion of death as dreamless sleep the most comforting. So I say that the atheist is motived to believe in death as dreamless sleep by fear, by comfort seeking.

    ReplyDelete
  41. I was reading Hume's Dialogues recently (very readable book, btw, would that all atheists wrote so entertainingly!)

    Anyway, I finally got to the bit where the characters discuss the First Way, and after reading the page, couldn't quite believe that Hume mis-represented it as much as Russell did later. It's almost feels like Hume brought up the straw-man intentionally, as an easy rhetorical means of dispensing with the argument.



    ReplyDelete
  42. >You mean Spinoza's view of God.

    Spinoza was a Pantheist. For him God and the Universe where the same thing.

    ReplyDelete
  43. "Putative psychological explanation: There is no relevant peer-directed social pressure placed upon naturalists or anti-theists to change their minds. So they know they can just phone it in and nobody will bat an eye, even if their arguments completely stink. For purely social reasons, the bar they have to clear is much lower than the bar raised for anti-naturalists and Theists."

    I think this is right on the money in many, if not most, cases.

    ReplyDelete
  44. Hence these writers would never say that “everything has a cause.” What they would say is that every actualization of a potential has a cause, or whatever is composite has a cause, or whatever has a feature only by participation has a cause, or whatever is contingent has a cause.

    It seems to me that this is a bit of a category problem. For category 1, we have the composite, participatory, contingent stuff and for category 2 we have non-composite, non-participatory, non-contingent stuff.

    Just to clarify, what besides the conclusion of the argument falls under category 2?

    ReplyDelete
  45. Hume's "Dialogues on Religion" was a very late creation, written when he had become far more interested in the vicissitudes and monetary profits of history. It is extremely literary where the motivations and points of view of each participant have to be sorted out. Scholars are not for sure, even, which participant really reflects Hume's 'real' point of view. This is also the period in which in one of his late essays he said the idea of an established church might not be a bad idea. People no longer know anything about the horrors of the English civil war which Thomas Hobbes wrote his contemporary quip about: "[In a state of war ] No arts; no letters; no society; and which is worst of all, continual fear and danger of violent death; and the life of man, solitary, poor, nasty, brutish, and short."
    Hobbes, Thomas. Leviathan.1651. Not only were there organized armies slaughtering everybody but "prophets" arose in every other village preaching a special creed that necessitated attacking the next village that did not accept it.
    -x-
    Ergo, for Hume, the "Dialogues" is not really a source of his philosophical conclusions but A TREATISE ON HUMAN NATURE 'might' be.He writes that book without “polite conversation” being his authorial “Golden Rule” whereas everything after that becomes more and more congenial, even at one point apologizing for certain rash anti-Christian remarks in his HISTORY OF ENGLAND where he also made an outstanding statement about the Church being a REAL force for peace when every crowned head at the time lusted for the glory of war [he hated Richard the Lionhearted].

    ReplyDelete
  46. To flat out say Spinoza was a pantheist is unjust until one has fully digested the ETHICS. And then, after that, one has to be extremely precise as to what a "pantheist" is.

    ReplyDelete
  47. @Robert Coble:

    Thank you for posting that link. Someone had posted such a link a few weeks ago, but I couldn't find it.

    ReplyDelete
  48. @Gary C. Moore:

    What is most important, in my opinion, is the fact that Hume was at bottom a philosophical amateur even though he was an excellent writer and a prodigious intellect.

    His historical writings are delightful, frequently insightful and very entertaining, even though they are seriously wrongheaded at points.

    But Hume's philosophical writings are a seriously mixed bag; he has enjoyed an inordinately inflated reputation and has had a very bad influence upon English speaking philosophy.

    His dialogues on natural religion are widely considered the coup de grace against natural theology even though he cites virtually nobody and his arguments draw no blood against titans like Aquinas and Aristotle.

    Following up upon my earlier comment: This cries out for special psychological investigation and not for serious philosophical engagement. Hume's central arguments have been hammered into the dirt but nobody seems to be paying attention.

    ReplyDelete
  49. For some reason the auto-fill signed my last comment as "U" even though it should have read "Untenured".

    ReplyDelete
  50. Perhaps my lack of philosophical training is showing itself here, but isn't Russell's use of the "everything has a cause" strawman particularly galling? He introduces his refutation of the cosmological argument with the acknowledgement that "cause" meant something much more robust in medieval philosophy than it does here in the 19th century, then refutes it in terms of the 19th century meaning. Doesn't he see how dishonest he's being?

    ReplyDelete
  51. I'm inclined to defend Hume on these matters; I think the real problem has less to do with Hume and more to do with how he is interpreted/used. He's often doing something very narrow and specific in focus that gets treated by people looking for a mascot as if it were sweeping and wide of scope. For instance, the bulk of the Dialogues is based on the assumption that there is one, and only one, possible argument for God's existence, and that this argument is sufficient for all religious purposes. The first assumption isn't arbitrary; you find Newtonian proponents of design arguments suggesting things that can at least be interpreted this way, and the Dialogues are Hume's contribution to Newtonian natural theology -- he concedes its basic principles, but argues that you can't get as much out of it as some of the major Newtonians thought you cuold. Likewise, you get weird interpretations of Hume as an atheist, despite the fact that Hume, in every text and even every recorded conversation on the topic that we have, always classifies himself as a theist, just a very skeptical one -- or as providing a general refutation of cosmological arguments, when he actually seems only to be arguing that Newtonians should not accept the arguments of Samuel Clarke. I think most of the problems with Hume trace back to similar attempts to interpret Hume as it suits the reader, rather than taking into account what Hume is actually doing.

    ReplyDelete
  52. For a venue where we is not guaranteed much sympathy the depth and rigour of Hume scholarship is disturbingly high (that’s meant as a complement). The problem is that, like some of the Pre-Socratics, Hume is a philosopher who is very much read to a formula and presented as such in text-books. I am inclined to agree with Untenured’s ultimate estimation of Hume (though think he was head-and-shoulders above the virtually all competition in Britain at the period) – Brandon’s comments on Design Arguments and Hume’s ultimate conclusions regarding them in The Dialogues are spot-on however.

    To do him justice I suspect Hume would pretty much hate the New Atheists and just consider them another, possibly more dangerous breed of ‘zealot’. Were he alive today he would be more David Stove than Christopher Hitchens.

    ReplyDelete
  53. Good Day to All,

    Since we are delving into the realm of why so many atheists often misrepresent the Cosmological Argument, then, if anyone is interested, I literally just posted my own lengthy blog post about this topic titled "Edward Feser Explains THAT the Problem Exists, but I Try to Explain WHY It Exists."

    Anyway, if you are interested, it can be found here:

    http://idontgiveadamnapologetics.blogspot.ca/2014/07/edward-feser-explains-that-problems.html

    Take care,

    RD Miksa
    www.idontgiveadamnapologetics.blogspot.com

    ReplyDelete
  54. I'll happily concede Brandon's point that the main problem with Hume has more to do with the uses he is put to than what he actually said.

    But the problem is still a big one: Hume clearly landed some hits on Clarke and Cudworth, but that is about as far as he goes in my opinion.

    But Hume doesn't land a single glove on Aquinas or Aristotle, even though he is regularly taken to have done so and is frequently assumed to have done so.

    I have taught the "Dialogues" ten times over, and if you can show me where Hume even approaches a sound argument against any of the Five Ways, properly construed, I would eat my hat if I owned one.

    Hume is wildly overrated, bottom line.

    ReplyDelete
  55. I've read that Spinoza angrily confessed in a letter of his that he wasn't a pantheist but a panentheist; he said something to the effect of "people keep saying that I equate god with nature when that couldn't be farther from the truth."

    Ah, it was a quote from the Wiki article on Panentheism. Not the best of sources, but the quote is taken from a book of correspondences of Spinoza.

    "as to the view of certain people that I identify god with nature (taken as a kind of mass or corporeal matter), they are quite mistaken"

    ReplyDelete
  56. Panentheism I am led to believe postulates the Universe is Divine but not the only extent of the Divine.

    I read once in Clawell Panentheism likens God's relation to the Universe as comparable to a Soul to a Body.

    Still Spinoza is not a Classic Theist so his "god" holds little interest for me other then as an object of academic study.

    ReplyDelete
  57. "Thank you for posting that link. Someone had posted such a link a few weeks ago, but I couldn't find it."

    That would have been me but it was over a month ago I think. I'm glad it has been so useful. Kudos goes to a fellow in the Thomism Discussion Group on Facebook who provided the article and link firstly.

    ReplyDelete
  58. @ dover-beach:

    The link to Fr. Clarke's essay was posted by you on 25 MAY 2014 at 6:54PM in response to Dr. Feser's post "This is Philosophy?".

    Thank you (again) for posting it!

    ReplyDelete
  59. A variation found in my daughter's school newsletter recently.
    Passionate Arguments

    One of the topics we look at in Senior School Christian Living touches on arguments - not "proofs" - for the existence of God. Arguments like these have existed for a very long time (pun not intended!), predominantly in Christian and Islamic contexts. Of course, students are convinced or unconvinced by them to varying degrees, while some sit on the fence.

    One such argument we look at is known as the “cosmological argument” because it argues for the existence of God based on the existence of the universe. One such version of it runs like this:

    (1) If the universe has an explanation then God exists

    (2) Everything has an explanation

    (3) The universe is a thing

    (4) Therefore, the universe has an explanation

    (5) Therefore, God exists

    Arguments like these serve as interesting exercises in critical thinking, of the very same kind that you would find in all the subject areas studied by Burgmann students. To that extent students learn that we can discuss and debate the topic of God rationally, just like we can discuss and debate the sciences, or literature, or politics.

    But, what else do these arguments have to offer our students?

    Surprisingly, I would suggest, that they have a unique capacity to engage the attitudes of our students’ hearts. I’ve found that whenever we discuss arguments like these in class, one rarely witnesses the cool and detached disposition that we normally associate with logical reasoning and critical thinking. Rather, a range of emotions are on display, either through speech or in body language, as students share their personal perspectives. It appears that whenever students choose to engage in serious, well-informed, critical thinking about God, it elicits a passionate response of some sort.

    Counterintuitively, the cosmological argument, which I think appears quite bland and sterile on paper, seems never to fail to fulfil one of the key aims of a quality Anglican education – that is, to attend to the attitudes of our students’ hearts toward God via their intellects.

    ReplyDelete
  60. So....you're saying everything has a cause? ;)

    ReplyDelete
  61. Please stop defending Hume.

    Hume was loaded with contradiction, duplicity and sophistry. His philosophy is simply scepticism somewhat veiled with what may well have been a deliberate strategy of failing to state the necessary logical consequences of his conclusions.
    I will never forget his call to burn what would amount to all classical philosophical works (I may be mistaken but I think this might be quoted in Dr. Feser's TLS).

    I haven't been able to read Hume without sensing an underlying hatred and animus of God or at least Christianity. There was also a strange tone of what seemed like an underlying effeminacy sometimes. But perhaps that was just a pathetic device to conceal the aforementioned hatred and animus and to make him seem fair, impartial, polite and conceal his utter contempt thereby shielding him somewhat from due angry criticism.

    Hume's contemporaries widely suspected him of being an atheist in disguise. I don't think we should dismiss their suspicion as being groundless. They may well have been in a position to know better than us.

    Hume can at least in part be thanked for many modern errors by his divorcing causes and effects in reducing them to chronological sequences; collapsing the intellect into imagination and even sensation; arguing something can come from nothing; inventing the fact-value dichotomy and no shortage of other absurdities beside.

    There is a difference between academic impartiality or neutrality and deliberately stripping an opponent without following it up by pointing out the fact that he is naked. To strip someone in public then take a step back and refuse to laugh or mock while the crowd does not make you any less guilty of exposing someone to ridicule simply because you didn't openly indulge in the subsequent ridiculing.

    Sure, Hume may not have openly laughed or stated in black and white what necessarily follows from his pseudo-analyses and investigations; notwithstanding, he left lady philosophy stripped naked before the eyes of the vulgar public again and again. Hume's philosophy reads more like clever and devastating propaganda than anything else.

    ReplyDelete
  62. Well Hume certainly disliked Christianity; there's no question about that (though his innate Toryhood and experience with the French atheists may have caused him to revise this for pragmatic purposes). The disturbing thing one has to bear in mind is that Hume begins his two main works with the assumption that metaphysics is useless, an assumption that his audience is presumed to share. Not that they would have applied it so overtly to theological topics as he did but I have a shrewd suspicion that many of Locke's otherwise pious disciples would have thought the same (classic philosophy is after all riddled with the dreaded 'Essence' word) – of course the same can equally be said about Kant, whom of course made no bones about his ultimate alliance to Theism.

    I make no defense of his philosophy, save for perhaps that the ‘Sceptical Realist’ interpretation should be considered more likely in the light of his correspondence. Ultimately he was an occasional philosopher of the type so prominent in that era. It would be interesting to know how his thought would have fared had not Utilitarianism developed afterwards.

    ReplyDelete
  63. Hume's contemporaries widely suspected him of being an atheist in disguise. I don't think we should dismiss their suspicion as being groundless.

    This is simply not true. It was not a widely held suspicion, although his rejection of revealed religion was known well enough, and he was suspected (rightly) of being a materialist in the sense of denying the immortality of the soul. Indeed, his actual atheist friends in France did not regard him as an atheist. And there is simply no evidence for it -- again, not only does he not ever classify himself as an atheist in his published writings, he doesn't in any conversation on the subject of which we have any evidence. It is quite literally a groundless aspersion.

    In general, attempts to paint him as a Big Bad are based on ignorance more than evidence. There is no fact-value dichotomy in Hume, for instance; the passages in which they can allegedly be found are in every case doing something else entirely (usually attempting to argue against Clarke and Malebranche, on their own principles).

    ReplyDelete
  64. I hope only slightly off-topic since he is the subject of the original post, but are there any Norris Clarke fans here. I've been thinking of adding him to the reading list, and looks like his books get pretty good reviews, but I haven't seem his discussed here much.

    ReplyDelete
  65. Sorry, that first sentence should end with a '?'.

    Are there any Norris Clarke fans here?

    ReplyDelete
  66. So I say that the atheist is motived to believe in death as dreamless sleep by fear, by comfort seeking.

    Try again.

    ReplyDelete
  67. I'm inclined to back up Brandon on his last comment but only partially.

    Hume, in his writings,always seemed vaguely open to the possibility that God existed but was always hesitant, but mainly because he was just generally skeptical and in a consistently inquisitive but hesitant manner.

    But you cannot, in my opinion, deny that Hume has done some serious philosophical mischief. I think its not unreaonable to paint him as "Big Bad", at least in terms of his influence. And mainly because his narrowly construed arguments against certain variants of natural theology have been taken up and run with, wildly explapolated from, and have contributed to the present anti-theistic consensus in Anglophone philosophy.

    ReplyDelete
  68. Whilst I don't like the way he seems to think random links settle arguments, and I think there are clear limits to the usefulness of trying to address motivations for beliefs, I'm inclined to agree with Step2 that, on balance, fear of death as the end is likely to outweigh fear of life after death amongst the population at large.

    Whilst atheists are far from immune to having reasons they want their atheism to be true, I think that the religious tend to have stronger reasons they want their faiths to be true.

    But I don't think this is a bad thing. It is completely fallacious to attack religious belief because of it, until you have shown such belief is not true. And, otherwise, what is wrong with wanting the better scenarios to be true, as long as it doesn't mean you do not examine your belief.

    ReplyDelete
  69. I think the anti-theistic consensus, such as it is, has very little to do with Hume, and a great deal to do with larger trends of cultural degradation in the twentieth century. For one thing, historically the interpretation of Hume as being such-and-such tends to lag behind the fashion of affirming such-and-such. It was so with the logical positivists, and so with the logical empiricists, and so with more modern forms of naturalism. And part of the problem is that Hume's influence, while considerable, is entirely an influence through dilettantes who pick and choose. In reality, we would all be better off if naturalists today were really Humeans; instead they claim to be Humean but are lying, not even being Humean in spirit, because they refuse to do what Hume does admirably: pursue the actual consequences of what is put forward. The anti-theists would be anti-theists anyway, because the number of people who have seriously been convinced to that position by serious engagement of Hume is necessarily extraordinarily small; would that Hume were more of an influence on them rather than less. The problem with naturalists today is not that they follow Hume but that they don't take him seriously enough.

    ReplyDelete
  70. @Step2

    The article doesn’t say what the ancients believed about death, or better, the afterlife. Clearly they had some such belief. How did those beliefs compare with the Christian version of the resurrection for comfort value? According to Hebrews, unfavorably. So I would conclude from this that death as dreamless sleep was not among the beliefs of a preponderance of the ancients in regard to the afterlife. The belief in death as dreamless sleep is more comforting even than the notion of the resurrection, with its possibility of damnation. The article does not address this comparison at all.

    ReplyDelete
  71. Brandon, I'll defer to you on this point because you know your Hume much better than I do. I'm just an analytic M&E guy who mines from the history of philosophy but does not specialize in it.

    But I second your point that contemporary naturalists are not consistent skeptics and do not follow their arguments wherever they lead. Their skepticism is mostly unprincipled and piecemeal. They tend to be "cafeteria skeptics."

    "Well, theism and dualism are obviously not true but modal realism is worth taking seriously, and I'm a realist about laws of nature and a realist about aesthetic properties too. etc etc"

    ReplyDelete
  72. @Step2

    Recently on a friend’s Facebook page, an atheist made this very argument for me, saying he had no fear of death at all, as there is, in his mind, nothing to it.

    ReplyDelete
  73. @Step2

    Death, strictly speaking, cannot be an experience. (For the Christian, death is real but temporary, for the atheist, real and permanent.) The article describes the presentation of images of “death” to atheists and to believers. But what could be an image of the experience of death? And if the images cannot be of the experience of death, what is really being tested? What images or associations do the priming images evoke? The consistent atheist should have no such images regarding the experience of death. But that is just not how the psyche works. Present it with an image and it will throw up associated images, memories, etc. The experiment conflates imagination with conception, I think, and so it misses its mark.

    ReplyDelete
  74. @Step2

    The article doesn’t specify what it means by images of “death.” Were any of the traditional images of hell included in the experiment as images of “death”? If not, why not? And wouldn’t that skew the experiment? (I can’t see how images of hell could be more comforting than “images” of the nothingness the atheist takes the afterlife to be.)

    ReplyDelete
  75. Untenured,

    There's also probably some effect just from standpoint -- as a HoP person, I tend toward a God's-eye point of view, so to speak, in which everything has its place, including error well-argued. And I look at Hume and I see not only his particular criticisms but also his Ciceronianism and contribution to virtue ethics, his genuine contributions to clarifying issues like the problem of the external world or the nature of good taste, etc., all bundled together with them. Someone without this way of viewing things might not have exactly the same assessment.

    ReplyDelete
  76. Brandon, that sounds right. The analytic M&E training I received actively militates against what you call "The God's-eye-view".

    It's all: "Give me the knock-down argument, and give it to me right now! In this very conversation! Appeals to historical context are just pure pluff!"

    There is a place for this hyper-critical mindset but it has definite and easily identifiable drawbacks.

    ReplyDelete
  77. In fact, as Clarke notes, Aquinas explicitly denies that everything has a cause. He held that “to be caused by another does not appertain to a being inasmuch as it is being; otherwise, every being would be caused by another, so that we should have to proceed to infinity in causes -- an impossibility…” (Summa Contra Gentiles II.52.5).

    Can anyone point to where Aquinas proves the bolded bit?

    Specifically, were does Aquinas prove the impossibility of an infinite regress?

    Thanks.

    ReplyDelete
  78. Contra gentiles I:13, 11-15; but it was Aristotle who proved it.

    ReplyDelete
  79. Clarke states, "One can, of course, use any meaning of 'cause' he wishes to. But one cannot mix two mutually exclusive meanings at once in the exposition of another's argument and then maintain that the argument does not hold."

    That's true. And it's the same sort of argument I've made regarding the evidence for the cosmological argument. No causal chain (motion or change) addresses "being itself." Therefore "cause" cannot be used to make the case.

    But let's ignore that problem because Clarke frames the issue as "sufficient reason." In so doing, he changes the words but not the error.

    He asserts, basically, that the world is not intelligible unless we find some "sufficient reason" for its existence. Just what that means is anyone's guess because to make his argument stick, he's forced to use "sufficient reason" in two mutually exclusive ways, switching from natural reason to supernatural reason.

    Is there "sufficient reason" to believe in gravity? We have plenty of (natural) empirical data that it exists. For most people gravity is intelligible. Yet why does it exist? We have no natural reason for that. Plain vanilla "sufficient reason" fails us here. So is gravity then unintelligible? If we have no "sufficient reason" for gravity's existence, are Newton's Laws also unintelligible? How could we derive "sufficient reason" for planetary motion based on a force for which there is no "sufficient reason?"

    That's the standard Clarke demands for nature as a whole. He's looking for a "sufficient reason" that has nothing to do with plain vanilla sufficient reason. He's looking outside the natural world, depending on supernatural cause, supernatural reason -- that is, insufficient reason.

    Clarke cannot wedge in a new "sufficient reason" that has nothing to do with the reasons we use in the natural world. He can't change the meaning in the middle of a proof. That's exactly what he expects the materialist to accept.

    Materialists (modern scientists, if you will) don't accept there are "sufficient reasons" for witches, vampires, ghosts, telepathic spoon bending or mind reading. Our "sufficient reason" precludes supernatural explanation. Clarke wouldn't think of asking us to accept the possibility that angels push the planets through their orbits. He'd be laughed at if he included this among his "sufficient reasons." Yet he wants us to forget the scope of our "sufficient reason" when it's convenient -- as a last ditch effort to slip one past us. Sorry, that simply will not work.

    Clarke complains that anti-theists have misunderstood the argument. But why have they done so? Maybe because the cosmological argument is otherwise too vague for comment, if not meaningless. I submit theists know this, which is why they usually depend on cause to build their case (hand-stick-stone, etc.). A materialist is not inclined to take Clarke's broad version of "sufficient reason" seriously. Clarke merely exposes a fundamental difference in epistemology. He gives me no "sufficient reason" to question mine.

    The fact is, his "sufficient reason" is capitulation. Plenty of people find "sufficient reason" to believe in ghosts. "Sufficient reason" can mean almost anything. Clarke ignores this crux of the matter ("Just how one arrives at acceptance of or commitment to this fundamental principle underlying all explanatory inquiry of any sort is not our present concern."). Therefore it's nonsense. It's nonsense to say a supernatural being provides its own "sufficient reason." I challenge anyone to make sense of that. I challenge anyone to show "sufficient reason" to stop asking "Why is there a God?"

    ReplyDelete
  80. There is really little reason to respond to this post save for the fact that this misunderstanding of the PSR has become all too prominent among philosophical dilettantes. The term ‘supernatural’ is entirely arbitrary in a philosophical context and can lend no support or opposition to an argument, to use it as such would be equivalent to one using the term ‘Holy’ to invalidate all criticisms of theological concepts. It is in, at least in this context, what I believe is termed a ‘thought-terminating cliché.

    For an entity to derive its PSR from itself means for it to possess modal necessity e.g. to exist in all possible worlds (to deny this point it arguably to deny all necessity which de facto ends up denying the laws of Logic such the PNC and thus is self-invalidating). Widely held examples of such necessaries are abstracta such as the objects of Mathematics, the content of Propositions or, more, controversially, Universals. Now since many philosophers, including a relatively large number of atheistic and thus one presumes Naturalistic thinkers, have endorsed such a view are we to conclude they are all ‘Supernaturalists’ and thus de facto wrong by arbitrary decree? Are we to call Quine a ‘Supernaturalist’ for his view on Sets?

    ReplyDelete
  81. Paul,
    First of all, the analogy to dreamless sleep is a poor one filled with misdirection since it implies a restful quality for a subject and an expectation of awakening. Unfortunately it has a famous pedigree as the backup argument for Socrates in the Apology. In other words, even if his idea of heaven didn't pan out death would still only be an eternal night of sleep and no cause for concern. A more accurate comparison is the end of the self as a subject. Granted that various legacies can provide limited reflections of the self as an object, the self-realizations of individual identity are gone. Second, your familiar tactic of “lies, damned lies, and statistics” normally has some merit but your categorical claim requires that statistics (and testing, and descriptions) be involved and in this case a personal anecdote does nothing to challenge the data.

    Here's a quote provided as a summary of its gist by one of the original trio who proposed TMT:
    “Life is tragic simply because the earth turns, and the sun inexorably rises and sets, and one day, for each of us, the sun will go down for the last, last time. Perhaps the whole root of our trouble, the human trouble, is that we will sacrifice all the beauty of our lives, will imprison ourselves in totems, taboos, crosses, blood sacrifices, steeples, mosques, races, armies, flags, nations, in order to deny the fact of death, which is the only fact we have.” – James Baldwin

    ReplyDelete
  82. @Step2
    “Second, your familiar tactic of “lies, damned lies, and statistics” normally has some merit but your categorical claim requires that statistics (and testing, and descriptions) be involved and in this case a personal anecdote does nothing to challenge the data.”
    I don’t recall mentioning statistics. And I don’t know what claim you mean.
    “First of all, the analogy to dreamless sleep is a poor one filled with misdirection since it implies a restful quality for a subject and an expectation of awakening.”
    Death cannot be an experience. One can either have no expectation of it, or expect “nothing” of it. In the former case, no, the analogy is not apt. But then also there would be nothing for an experiment to detect, nothing for any psychology to study. I put “nothing” in quotes to indicate that in the latter case it cannot be taken literally. Having an expectation, one must expect *something* even if that something is only an awareness of darkness, silence, or numbness. Here the analogy to dreamless sleep is apt.

    You, and the article you cited, have claimed, without *argument*, that death is terrifying. I have given arguments, I think sound, for why it is not, or at least for why I do not find it terrifying. The article came close to saying that anyone who is at peace with death is lying to himself, is in “denial.” Here I think Popper’s criticism of such *irrefutable* theories as unscientific, is valid.

    ReplyDelete
  83. Step2,

    Here's a quote provided as a summary of its gist by one of the original trio who proposed TMT:

    Oh dear. Someone is in need of a history lesson. I'm about to leave to meet with my wife, so I haven't the time just now. But maybe this is just as well --it'll give you time to shore up whatever defenses may be available to your for the purpose of protecting you assertion. ;)

    ReplyDelete
  84. (s/b "...to you...")

    ReplyDelete
  85. I don’t recall mentioning statistics. And I don’t know what claim you mean.

    The claim that atheists are, compared to theists, motivated by comfort seeking beliefs. Any attempts to study this phenomenon and to demonstrate a contrast are naturally going to be statistical. There isn't a one-size-fits-all for religious belief or for atheistic beliefs. But in order to support your categorical claim you have to examine them as best you can through statistical surveys. When you do so you find that in general religious beliefs do provide greater comfort in the face of mortality than not having those beliefs. How is that difficult to understand?

    ReplyDelete
  86. @Step2:

    Just FYI, the "original trio who proposed TMT" were Jeff Greenberg, Sheldon Solomon, and Tom Pyszczynski. That James Baldwin quote was not offered as the "gist" of TMT, although some have invoked TMT in his support (or him in its).

    ReplyDelete
  87. Scott,
    Here. Fourth page, first column.

    ReplyDelete
  88. Oh, I see what you mean -- Jeff Greenberg provided the Baldwin quote as an apt summary of TMT. I thought you were saying James Baldwin himself was one of the three original proponents of the theory.

    ReplyDelete
  89. @Step2

    I said they were motivated by comfort seeking, not by comfort seeking beliefs. They, atheists, believe what they believe about death because it is the most comforting thing to believe.

    “But in order to support your categorical claim you have to examine them as best you can through statistical surveys.”

    No, I don’t. I can offer arguments and see how they fare in the course of the dialectic. And I can at least question the validity of the study, especially when I see signs that the study is not properly scientific, like the assertion of irrefutable claims such as “Those who do not fear death are in denial.” (Please note, that’s a paraphrase, not a direct quotation.)

    But let’s suppose the study is valid. How, specifically, does that affect my argument?

    ReplyDelete
  90. Step2 is a troll. Ignore him.

    ReplyDelete
  91. With apologies to Anonymous, I'll now follow up on my earlier comment..

    ReplyDelete
  92. Step2,

    Scott said, "I thought you were saying James Baldwin himself was one of the three original proponents of the theory." And that's just what I had thought. The joke, apparently, is on me. (And I actually did laugh over my misreading when I finally saw the intended meaning of your "one of the original trio" statement.)

    Still, although the correct composition of "the original trio who proposed TMT" was in mind when I made my "history lesson" comment, the correct composition was but a minor point. The major point had to do with the "larger picture" going back to your earlier link to Richard Beck's article regarding Christians and TMT.

    Nonetheless, given your now known familiarity with the Greenberg and Arndt article, it seems obvious that no "history lesson" is needed, as you no doubt recall that that article makes clear that "a decent case can be made that TMT is an ancient theory that can be traced back to one of the first narrative texts from around 3000 BC, the Sumerian Epic of Gilgamesh," and that "the first person to put the basic points of TMT together seems to have been the famed Greek historian Thucydides."

    - - - - -

    Some quotations from Josef Pieper's The Four Cardinal Virtues: Prudence, Justice, Fortitude, Temperance:

    o Fortitude presupposes vulnerability; without vulnerability there is no possibility of fortitude. (p. 117)

    o Because man is by nature vulnerable, he can be brave. (p. 117)

    o All fortitude stands in the presence of death. (p. 117)

    o To be brave is not the same as to have no fear. Indeed, fortitude actually rules out a certain kind of fearlessness, namely the sort of fearlessness that is based upon a false appraisal and evaluation of reality. (p, 126)

    o Fortitude presupposes in a certain sense that man is afraid of evil; its essence lies not in knowing no fear, but in not allowing oneself to be forced into evil by fear, or to be kept by fear from the realization of good. (p. 126)

    ReplyDelete
  93. @Scott and TheOFloinn

    I looked up the passage, in Contra Gentiles, that you suggested.

    The reasoning used by Aquinas to support the premise that an infinite regress is an impossibility requires an Aristotelian understanding of Physics.

    Do you guys find this reasoning to be sufficient to support the contention that an infinite regress is, in fact, an impossibility?


    ReplyDelete
  94. Bob,

    No, it doesn't require an Aristotelian understanding of physics. Aquinas is referring only to metaphysical arguments based on the most general observations about change in the world. Dr. Feser defends these premises, like the impossibility that there is no first mover in a per se causal chain, time and time again.

    ReplyDelete
  95. @ Jeremy Taylor

    Your comment is factually mistaken.

    Have you actually read the passage to which Scott and TheOFloinn referred me?

    ReplyDelete
  96. @ Jeremy Taylor

    Here is the passage in question:

    [11] The second proposition, namely, that there is no procession to infinity among movers and things moved, Aristotle proves in three ways.

    [12] The first is as follows [VII, 1]. If among movers and things moved we proceed to infinity, all these infinite beings must be bodies. For whatever is moved is divisible and a body, as is proved in the Physics [VI, 4]. But every body that moves some thing moved is itself moved while moving it. Therefore, all these infinites are moved together while one of them is moved. But one of them, being finite, is moved in a finite time. Therefore, all those infinites are moved in a finite time. This, however, is impossible. It is, therefore, impossible that among movers and things moved one can proceed to infinity.

    [13] Furthermore, that it is impossible for the abovementioned infinites to be moved in a finite time Aristotle proves as follows. The mover and the thing moved must exist simultaneously. This Aristotle proves by induction in the various species of motion. But bodies cannot be simultaneous except through continuity or contiguity. Now, since, as has been proved, all the aforementioned movers and. things moved are bodies, they must constitute by continuity or contiguity a sort of single mobile. In this way, one infinite is moved in a finite time. This is impossible, as is proved in the Physics [VII, 1].

    [14] The second argument proving the same conclusion is the following. In an ordered series of movers and things moved (this is a series in which one is moved by another according to an order), it is necessarily the fact that, when the first mover is removed or ceases to move, no other mover will move or be moved. For the first mover is the cause of motion for all the others. But, if there are movers and things moved following an order to infinity, there will be no first mover, but all would be as intermediate movers. Therefore, none of the others will be able to be moved, and thus nothing in the world will be moved.

    [15] The third proof comes to the same conclusion, except that, by beginning with the superior, it has a reversed order. It is as follows. That which moves as an instrumental cause cannot move unless there be a principal moving cause. But, if we proceed to infinity among movers and things moved, all movers will be as instrumental causes, because they will be moved movers and there will be nothing as a principal mover. Therefore, nothing will be moved.

    ReplyDelete
  97. When you said physics, did you mean physics the discipline or Physics, Aristotle's work?

    In current terminology, what you quote, as far as I know, is metaphysics, not physics, even if it comes from Aristotle's work called Physics.

    I'm not a hundred percent sure, because in the Aristotelian framework there are intermediaries between metaphysics and natural science (under which what call physics would be located), such as philosophy of nature. But I believe motion, or change, in its most general sense falls under the category of metaphysics.

    What is most essential to underline, though, is nothing that you quote shows that the argument Aquinas is making relies on Aristotelian physics in the sense that we mean physics (where Aristotle's doctrines are obsolete for the most part). The argument just relies on some basic philosophical insights.

    The ironic thing is this is you are discussing something that Dr. Feser talks about again and again in his works.

    ReplyDelete
  98. The reasoning used by Aquinas to support the premise that an infinite regress is an impossibility requires an Aristotelian understanding of Physics.

    The first argument in SCG does, because it's nothing other than a cleaned-up version of Aristotle's. (Although it's worth noting that everything actually appealed to in the first argument would have been perfectly acceptable at least as late as the nineteenth century.) But the second and third are clearly arguments that infinite regress is contradictory for certain classes of causes.

    ReplyDelete
  99. @ Jeremy Taylor

    On the contrary, it seems fairly clear that Aquinas is relying on the proof that Aristotle provided with regards to the impossibility of an infinite regress. The proof is clearly based on Aristotle's understanding of physics.

    Of course, my question was simply if asking if you found the Aristotelian proof convincing.


    ReplyDelete
  100. @Brandon

    But the second and third are clearly arguments that infinite regress is contradictory for certain classes of causes.

    I noticed that to, but upon reflection I realized that even those relied, in principle, on Aristotle's physics.

    ReplyDelete
  101. the proof that Aristotle provided with regards to the impossibility of an infinite regress ... is clearly based on Aristotle's understanding of physics.

    Physics ain't in it. It's a logical proof that a series of instrumental causes cannot regress without limit. An instrumental cause has no causal power unless it is being used as an instrument by another. If there is no primary cause, then the secondary causes are inert, like a piano without a pianist.

    ReplyDelete
  102. I noticed that to, but upon reflection I realized that even those relied, in principle, on Aristotle's physics.

    It's utterly unclear what you mean by this; to what principle, precisely, do you refer?

    ReplyDelete
  103. @TheOFloinn

    Can you clarify the logical syllogism for me? I don't see it.

    ReplyDelete
  104. Neither of them refers to or draws on any logical property of simultaneity.

    ReplyDelete
  105. It should be pointed out, as well, that even when arguments do (as with the first) this is not "relying on Aristotle's physics" since a simultaneity account of causation is not in any way exclusive to Aristotle.

    ReplyDelete
  106. @Brandon

    Yes, they do rely on simultaneity. explicitly so.


    The proof Aquinas points to does rely on Aristotle's physics, per Aquinas's own words.

    ReplyDelete
  107. No, they don't; you're misreading them.

    Your second sentence is an obvious equivocation on what it means to rely on a physics.

    ReplyDelete
  108. @Brandon

    No equivocation. Read the references provided by Aquinas.

    And I am not misreading.

    it is necessarily the fact that, when the first mover is removed or ceases to move, no other mover will move or be moved.



    ReplyDelete
  109. I've read them on multiple occasions including in the original Latin, and compared them with Aquinas's discussions in his commentary on the Physics and elsewhere, Bob. But it's irrelevant; to say that 'it relies on Aristotle's physics' can only mean in your original question 'it relies on Aristotle's physics', not 'it relies on a purely logical argument that Aristotle happens to use in his book called the Physics'.

    You are indeed misreading; the premise relies on complementarity of cause and effect, not simultaneity, and there is no account of cause and effect that does not imply that cause and effect are complementary, because it is nonsense to say that something is a cause without there being anything it is a cause of, or that it is an effect that is not an effect of anything.

    ReplyDelete
  110. @Brandon

    This:

    it is necessarily the fact that, when the first mover is removed or ceases to move, no other mover will move or be moved.

    is factually incorrect. It is based on an understanding of physics (Aristotelian) that has since been falsified.

    The understanding of cause and effect used in this argument does, in fact, require simultaneity. This causes (pun) the argument to fail.

    ReplyDelete
  111. No, Bob, as I said before, it is a logical property following from something being identified as a cause or effect: cause and effect are complementary. This is the direct implication of the Latin word translated here as 'removed'. The logical property used in the argument does not require simultaneity; nor does simply repeating yourself make it otherwise.

    ReplyDelete
  112. @Bob

    Simultaneity, according to Professor Feser, is not what the proof relies on. Scott provided a link for me to an article of Ed's which I'll try to find.

    ReplyDelete
  113. Daniel,

    "For an entity to derive its PSR from itself means for it to possess modal necessity e.g. to exist in all possible worlds (to deny this point it arguably to deny all necessity which de facto ends up denying the laws of Logic such the PNC and thus is self-invalidating)"

    When someone speaks of "necessity" in "all possible worlds" I know they're not serious. Yes, I do deny necessity of this sort. Does this refute gravity in this world? Of course not. Get back to me when you have achieved your deification. I'm anxious to hear tales from all those possible worlds.

    Anyway, for you to implicitly compare the supernatural to the logical is unconvincing, to say the least.

    ReplyDelete
  114. @Bob

    Here's the link.

    http://edwardfeser.blogspot.com/2010/08/edwards-on-infinite-causal-series.html

    ReplyDelete
  115. it is necessarily the fact that, when the first mover is removed or ceases to move, no other mover will move or be moved.

    If the pianist is removed or ceases to play, the piano will cease to move. (It does not instantaneously cease. Everyone knew that if you disengaged the crown gear, the mill stone would continue to roll for a short time, due to the impetus [momentum]. See "first and last moments.")

    The theorem does no apply to accidentally ordered series. In those cases, Cause i has its causal power regardless whether Cause i-1 continues to act upon it. Father begetting son is the classical example, but toppling dominoes will do as well. Aquinas distinguishes this case and allows as it might regress indefinitely.

    "Concurrency" is a better term than "simultaneity," since the latter implies dimensionless points of time. As Whitehead points out in his Principle of Relativity, physical events are not point-events, but always have a duration.

    ReplyDelete
  116. @TheOFloinn

    What causes the mill stone to stop is not actually the disengaged crown gear.

    That is what is being missed in this conversation.

    ReplyDelete
  117. What causes the mill stone to stop is not actually the disengaged crown gear.

    That is what is being missed in this conversation.


    This is not what is being missed your conversation, since it is irrelevant. What causes the millstone to stop is obviously going to be the intervention of another cause. This is irrelevant to the question of what causes the millstone to be moving.

    ReplyDelete
  118. @Brandon

    Yet it is relevant to this:

    it is necessarily the fact that, when the first mover is removed or ceases to move, no other mover will move or be moved.

    ReplyDelete
  119. Again, no, it's not; that's precisely the point TOF was making. And again, it doesn't magically become relevant just because you repeatedly say it does in the face of the actual evidence about what the text says and what the scholastics actually about these matters.

    ReplyDelete
  120. What causes the mill stone to stop is not actually the disengaged crown gear.

    If the crown gear remains engaged, the mill stone will continue to turn because it is by virtue of the crown gear that motion is given to the millstone. The mill stone does not possess this motion per se, that is: in itself. (Neither does the crown gear, but let's not chase it back.) The question is what gives motion to the mill stone (or the piano), not what causes it eventually to stop once that impetus is removed. The mill stone does not move at all* unless the crown gear gives it motion. Once the crown gear is disengaged, the mill stone returns to its natural state of quiescence. Buridan observed that it did so gradually, but the "kinesis of stopping" begins with the removal of the motive power.

    (*)To forestall the usual sophistry: yes, the millstone is moving because it rides aback a rotating and revolving earth whirling through a galaxy flying through space. But that don't grind no corn.

    ReplyDelete
  121. It's perhaps worth adding, in addition to the fact that the Latin terms used in the argument don't automatically require temporal process (being usable also for talking about purely mathematical and logical situations or for constructing hypothetical situations) that there are several cases in which Aquinas is clear that cause and effect are not simultaneous in the way we usually mean simultaneity -- e.g., generators as the movers of things exhibiting natural motion, or the causing of the motion of the primum mobile. As TOF notes, even when the scholastics do talk about simultaneity, it can cover other things than what we would usually call simultaneity, and thus we cannot import without any qualification our assumptions into the terms of the arguments.

    ReplyDelete
  122. @ Don Jindra:

    You seem to be deeply confused about the meaning of the term "sufficient reason". In the PSR context at least, it doesn't mean "sufficient reason for believing in something", but rather "explanation for why something is the way it is". The fact that objects fall to the ground might provide us with a sufficient reason for believing in the existence of gravity, but it doesn't provide an explanation for why the force of gravity exists, why it affects things in the way it does, and so on.

    ReplyDelete
  123. @msgrx,

    From the Stanford Encyclopedia of Philosophy:

    "Do all facts—including the most ordinary ones—demand an explanation? If you accept the Principle of Sufficient Reason (= PSR), you will require an explanation for any fact, or in other words, you will reject the possibility of brute, or unexplainable, facts."

    I stand by my "deeply confused" understanding of PSR. Plus I already mentioned that "why gravity" is an issue.

    ReplyDelete
  124. @Msgrx
    Wish I’d said that.

    @Don Jindra

    A couple definitions of the PSR (principle of sufficient reason).

    “[…] PSR states that “everything is intelligible” and that “there is sufficient reason or adequate necessary objective explanation for the being of whatever is and for all attributes of any being […]” from *Scholastic Mataphysics* by Edward Feser page 137-8

    Or

    “The adequate and necessary objective explanation of something.” - Dictionary of Scholastic Metaphysics

    I not yet familiar enough with the various proofs, cosmological or otherwise, to defend them or really even to explain them. So maybe I should just be quiet. It’s just that I too found your presentation of “sufficient reason” confusing.

    ReplyDelete
  125. Not sure this has been linked already:

    http://edwardfeser.blogspot.com/2011/02/can-we-make-sense-of-world.html?m=1

    ReplyDelete
  126. I’m still trying to understand causal series ordered *per se*. If “first” doesn’t have to mean first in time, what could it mean? How about first in importance? The thing upon which every other member of the series depends is first in importance. The next question is, “Depends how, for its existence, its motion, what?” St Thomas’s examples involved motion, change. Change is movement from potentiality to actuality. So the least independent thing would have to be the least actual, namely prime matter. For some reason I think of the ocean. Everything else in the ocean depends on the water, and that simultaneously. I can’t fill in the whole hierarchy. I know I’ve made some skips here but…

    ReplyDelete
  127. You do know Don Jindra is a massive troll. His modus operandi is to make half-baked criticisms, ride them until they fall apart, then duck away or just move onto yet another half-baked criticism.

    He was banned from coming here in the past for repeated trolling.

    ReplyDelete
  128. @Paul Amrhein:

    "If 'first' doesn’t have to mean first in time, what could it mean?"

    It means ontologically or logically first. God, as Prime Mover, isn't part of the series of (secondary) causes, but outside it, as the (primary) cause of the entire series.

    ReplyDelete
  129. @Scott

    To be ontologically “second” means to be less actual in relation to a “first” thing. Any number of “second” things may stand simultaneously in such a relation to a single “first” thing - one agent, many patients. Yes?

    Professor Feser, in *Aquinas* mentions that there is no “third”or etc. in such a series (page 72). I think I’ll find it helpful to keep that in mind. When I think of it that way, the whole problem of going backwards through causal chains seems to disappear.

    ReplyDelete
  130. @Paul Amrhein:

    "To be ontologically 'second' means to be less actual in relation to a 'first' thing."

    Or more dependent. God would still exist if the series didn't, but the series wouldn't still exist if God didn't.

    "Any number of 'second' things may stand simultaneously in such a relation to a single 'first' thing - one agent, many patients. Yes?"

    Yes. In fact this is precisely the situation with God and the things in the world.

    ReplyDelete
  131. @Paul Amrhein,

    Those definitions are similar to the Stanford definition I already quoted so I don't get your point. My basic point is that Clarke replaces (or must replace) "sufficient reason" with a subjective standard which he merely labels "sufficient reason." His standard may satisfy him personally but it fails with those of us who do demand objective reasons. The difference is in epistemology and that defines our standard for our "sufficient reasons." IOW, some of what Clarke sees as objective I see as subjective.

    @Anonymous,

    >>You do know Don Jindra is a massive troll. His modus operandi is to make half-baked criticisms, ride them until they fall apart, then duck away or just move onto yet another half-baked criticism.<<

    That's maybe 10% true. Anyone who was around three years ago knows I've never ducked away from an argument. I've always been doggedly persistent in defending my positions and I know of no case where those on this blog were able to prove me wrong. Of course those here think a "materialist" like me has a half-baked POV. That goes without saying. But I think their's is half-baked -- like Clarke's above. So where do such accusations get us?

    I was "banned" because I noted the political implications of Feser's philosophy. It's always my hope that someone confident of the truth of his position will defended his position rather than trying to silence the other. I could say more but it would be imprudent.

    ReplyDelete
  132. @Don Jindra

    “Those definitions are similar to the Stanford definition I already quoted.”

    Sorry, I guess I just missed that, didn’t go back far enough.

    Can you give an example of something Clarke sees as objective but you see as subjective? (Maybe you’ve already done this too but please bear with me.) Maybe, for instance, you see universals as subjective, where the Aristotelian realist would think of them as objective, or mind-independent.

    ReplyDelete
  133. @Don Jindra

    “Is there "sufficient reason" to believe in gravity? We have plenty of (natural) empirical data that it exists. For most people gravity is intelligible. Yet why does it exist? We have no natural reason for that. Plain vanilla "sufficient reason" fails us here. So is gravity then unintelligible? If we have no "sufficient reason" for gravity's existence, are Newton's Laws also unintelligible? How could we derive "sufficient reason" for planetary motion based on a force for which there is no "sufficient reason?"

    What is it for sufficient reason to fail? Wouldn’t that mean showing that there is some thing that can have no explanation? Who has shown that gravity can have no explanation? (I recall a letter to the editor from a local physics professor correcting an earlier letter from a layman claiming that gravity was unexplained. He didn’t go into details. Anyway that’s really beside the point.)

    Also, where in the definitions of sufficient reason is it said that explanation must be “natural?”

    ReplyDelete
  134. Paul,
    But let’s suppose the study is valid. How, specifically, does that affect my argument?

    It only means you should qualify it so it isn’t a blanket statement that applies categorically to all atheists and theists.

    Bob,
    Aristotle’s physics and metaphysics are quite strange compared to modern physics and cosmology. This should make it difficult to translate it into modern terms without altering at least some of his conclusions. It also seems to me like his problems may go beyond that and his principles were inconsistently applied. For example, it is unclear to me whether or not Aristotle thought the Primary Unmoved Mover (which inspires the other unmoved movers of the celestial spheres) is an actual infinity, which is relevant because in every other context he considered actual infinity impossible.

    ReplyDelete
  135. @Step2

    Okay let’s see. Here’s the original.

    They, atheists, believe what they believe about death because it is the most comforting thing to believe.

    Here’s a more qualified version.

    Many atheists, I think, believe what they believe about death because it is the most comforting thing (for them?) to believe. Why should they not be as prone to “motivated reasoning” as anyone else?

    I agree that it should be qualified. I also do not wish to *ignore* empirical studies which may provide counterexamples. (The study did (does) give me pause.)

    ReplyDelete
  136. Paul,
    Your qualification is reasonable and fair. Thanks.

    ReplyDelete
  137. @ Step2

    The specific point that seems most relevant, that the entire proof rests upon, revolves around this statement:

    it is necessarily the fact that, when the first mover is removed or ceases to move, no other mover will move or be moved.

    Despite the arguments that have so far been put forward in defense of this statement, I remain unconvinced that this statement is actually true.

    ReplyDelete
  138. As you know Bob,
    a clarinet does not play itself
    and once the clarinetist ceases to play, the music stops.

    ReplyDelete
  139. This comment has been removed by the author.

    ReplyDelete
  140. @TheOFloinn

    Existence is not, however, a clarinet.

    ReplyDelete
  141. @Paul Amrhein,

    "where in the definitions of sufficient reason is it said that explanation must be 'natural?'”

    Nowhere. PSR doesn't tell us what reasons are reasonable. IMO, only 'natural' reasons are reasonable. Clarke has broader notions of reasonableness. This is one reason why his essay fails for me. PSR might make something intelligible to him but not necessarily to me. Additionally, PSR tells us nothing about truth.

    For example, Clarke points us to his article, "Analogy and the Meaningfulness of Language about God." I couldn't find that, but I found a reply to Kai Nielsen concerning it. Clarke imagines a higher alien intelligence. How might they communicate ("opening up an analogous concept beyond [our] presently known range of examples")?

    "We decide we should remain prudently open... -- [communication] would not even have to be through material signs but might be by direct telepathy or thought-communication" We have no idea "what the mode of intelligence involved might be like or how it might function in itself, even when not attached to a body. Yet it makes perfect sense, and in the concrete it is quite easy...to open up the range of meaning of what we now experience and understand as intelligence to include in expectancy some possible level at present quite unknown and uncharacterizable by us."

    IOW, body-less telepathy is "intelligible" to Clarke. But this subjective intelligibility doesn't imply telepathy (or revelation) is indeed a fact. PSR does not imply fact. Likewise, if Clarke reduces the cosmological argument to PSR, it does not imply fact.

    Fundamentally it boils down to our (subjective) epistemology, not PSR.

    ReplyDelete
  142. "Intelligible in itself" isn't the same thing as "intelligible to us." I believe I linked the relevant blogpost a few comments back.

    ReplyDelete
  143. "Existence is not, however, a clarinet."

    You're just avoiding the issue, that "a series of instrumental causes cannot regress without limit. An instrumental cause has no causal power unless it is being used as an instrument by another. If there is no primary cause, then the secondary causes are inert,...." In the example the OFloinn provides, existence is the music, the clarinetist is the Primary Cause of the music,which is categorically different from all secondary causes.

    ReplyDelete
  144. Don Jindra: I've always been doggedly persistent in defending my positions and I know of no case where those on this blog were able to prove me wrong.

    Well, that's true. DJ is handily proven wrong in every case, but he does not know it. He has a remarkable track record of missing the point. The problem isn't that he gets things wrong — we all gets things wrong — but his, er, dogged persistence means that the more he paints himself into a corner, the more he misses the point. Anyone who's been here a while knows that all attempts at conversation with him are utterly unproductive.

    I was "banned" because I noted the political implications of Feser's philosophy.

    How amusing. Feser doesn't care whether comments agree with him or not, only whether they contribute something constructive (and even then he's pretty generous about drawing the line — I'd be rather more strict in my pruning). But all the comments on old posts are still there for anyone who wants to see for himself that posts have to be nothing more than verbal vandalism for Feser to block them.

    ReplyDelete
  145. Bob: Despite the arguments that have so far been put forward in defense of this statement, I remain unconvinced that this statement is actually true.

    It's fairly obvious, but my guess is that you are getting confused by all the different motions going on. When you shove a millstone, the motion or change is that of imparting momentum to the millstone. When the millstone moves, or continues to move after that, the motion is change of position owing to momentum possessed by the millstone. Something that has momentum will continue to change its position until acted upon by some force (that's what momentum is). Something that does not have momentum will not suddenly get it out of nowhere. That's as true for modern physics as for the Aristotelian variety. Indeed, the continued motion of the millstone in empty space is the very same principle at work: the millstone will not undergo change (such as losing momentum it already has) without some cause (e.g. friction) continuing to act on it. Removing the pushing cause, and the effect of getting more momentum does indeed cease; remove the retarding cause (friction), and the effect of losing momentum will likewise cease.

    ReplyDelete
  146. @Don Jindra

    “Nowhere. PSR doesn't tell us what reasons are reasonable. IMO, only 'natural' reasons are reasonable. Clarke has broader notions of reasonableness. This is one reason why his essay fails for me. […] ”

    Understood. I think you actually believe in a restricted version of PSR - “Everything has a natural explanation.” No?

    ReplyDelete
  147. In the example the OFloinn provides, existence is the music, the clarinetist is the Primary Cause of the music,which is categorically different from all secondary causes.

    Doesn't this analogy make God the efficient cause of being rather than its final cause?

    ReplyDelete
  148. @Step2:

    "Doesn't this analogy make God the efficient cause of being[?]"

    Yes.

    "…rather than its final cause?"

    …and no. The two aren't mutually exclusive, and the illustration isn't intended to illustrate anything about final causes.

    ReplyDelete
  149. Scott,
    Aristotle rejected the idea of God as efficient cause since efficient causes require an action-reaction, in other words a moving mover.

    ReplyDelete
  150. @ dover-beach

    No, I understand the analogy that TheOFloinn was trying to make. I do not think that it is a good one.

    A more proper analogy would be a case where the existence of the clarinet itself is predicated on the clarinetist playing it. That once the clarinetist pauses, the clarinet itself ceases to exist.

    That would be a more accurate analogy and goes directly to my objection.

    ReplyDelete
  151. @Step2:

    "Aristotle rejected the idea of God as efficient cause…"

    …and Aquinas didn't. And?

    ReplyDelete
  152. @Bob:

    "A more proper analogy would be a case where the existence of the clarinet itself is predicated on the clarinetist playing it.…That would be a more accurate analogy and goes directly to my objection."

    Your original objection had nothing to do with "existence." Your objection was to Aquinas's statement that "when the first mover is removed or ceases to move, no other mover will move or be moved." In what relevant way does the clarinet continue to "move" (i.e. change) when the clarinetist stops playing it?

    ReplyDelete
  153. @Scott

    Of course it has to do with existence because that is precisely what is meant to be implied by the analogy.

    ReplyDelete
  154. @Bob:

    "Of course it has to do with existence because that is precisely what is meant to be implied by the analogy."

    I'm not sure what this statement is supposed to mean, but there was no "analogy" in your original objection:

    The specific point that seems most relevant, that the entire proof rests upon, revolves around this statement:

    it is necessarily the fact that, when the first mover is removed or ceases to move, no other mover will move or be moved.

    Despite the arguments that have so far been put forward in defense of this statement, I remain unconvinced that this statement is actually true.


    And if you're referring to TheOFloinn's "analogy" (really an illustrative example) in direct reply to that objection, then I don't know why you take it to be about "existence" when your objection wasn't. TOF's illustration is pretty obviously of a series that stops "moving" (changing) when its first member ceases to act through it.

    ReplyDelete
  155. @Paul Amrhein

    "I think you actually believe in a restricted version of PSR - “Everything has a natural explanation.” No?"

    More or less. I believe natural explanations are the only reasonable explanations -- the only ones I find compelling.


    @Anonymous,

    "'Intelligible in itself' isn't the same thing as 'intelligible to us.'"

    It definitely is the same thing. As far as we know we are the only beings who ask if something is 'Intelligible in itself' and we definitely ask it from our perspective, that is, Do we consider it 'Intelligible in itself'?


    @Bob,

    You are exactly right and I've made the same point many times. The violin analogy fails because it's not about existence of anything, including the violin. Plus, the music does not stop when the player stops. TheOFloinn is wrong about that. The music keeps traveling at the speed of sound. Its effects go on through the end of time even though it gets dispersed into an unrecognizable form within a few miles. Any cause starts an infinite chain of cause and effect that never ceases. Never. Even another cause cannot stop the earlier cause because in stopping it, it itself is affected by that original cause. It does not matter what happens to the original violin player. (And I should mention even he is an effect.) This violin analogy is exactly what Clarke is complaining about in the 'Curious Blindspot' essay ("One can, of course, use any meaning of 'cause' he wishes to. But one cannot mix two mutually exclusive meanings...") The violin analogy mixes those two kinds of causes. As far as we know, causes related to motion and change are not at all similar to causes related to being itself. The sad fact is, we have no knowledge of causes related to being itself. So no relevant analogy is possible.

    ReplyDelete
  156. @Scott,

    If it is not about existence, than the analogy is simply wrong factually.

    ReplyDelete
  157. As Scott notes, existence is irrelevant here -- dragging it in is a conflation of different kinds of causations; the question at hand was specifically about moving causes, which are causes of changing, not of existence as such; and as Mr. Green noted, one must be careful not to equivocate on the other end about which change is in view when talking about a particular causal situation.

    ReplyDelete
  158. @Bob:

    What Brandon said. Moreover, it ought to worry you a little that you have Don Jindra on your side. I won't reply to him directly as he's been banned from the site, but for your own edification I'll point out that this:

    "[T]he music does not stop when the player stops. TheOFloinn is wrong about that. The music keeps traveling at the speed of sound. Its effects go on through the end of time even though it gets dispersed into an unrecognizable form within a few miles."

    …is a perfect example of what Mr. Green meant by getting confused about what change is in view. Whatever may happen with the sound waves that "carry" the music that has already been played, the point is that no new music is being generated after the (voilin or clarinet) player stops playing.

    ReplyDelete
  159. @Don Jindra

    Please consider the following an exercise in “writing to learn” on my part. It’s my attempt to explain the cosmological argument, in part. I want to see if you find anything “supernatural” in its premises.

    A deck of cards is an actual thing “containing” many potential things. It is partly potential, partly actual. One of those potential things is a full house. So we say that the full house is “in potency.” Now a potential thing cannot change itself into an actual thing, cannot “move” itself. Although our deck of cards is partly actual, its actuality does not include the capacity to change or to move itself. So if our potential full house is to change into a real full house, it must be changed by some actual thing with the relevant causal power.

    If the universe is like our deck of cards in being a mixture of act and potency lacking the power to realize its own potentials, then…what? Its potentials must be realized by something outside it.

    The next steps would have to do with the nature of that thing. But I’m not ready for that yet.

    PS I didn’t know you’d been banned from the site. I’m going to follow the rules of the site. But you’re welcome to contact me by other means. I’m easy to find.

    PPS I know the proof at issue was about the PSR. I just want to see if you might agree that some version of the proof needn’t involve the supernaltural in its premises.

    ReplyDelete
  160. …and Aquinas didn't. And?

    Did Aquinas provide any reasons Aristotle was wrong about efficient causes to justify this glaring exception? If not then Aquinas was inadvertently ascribing change to God.

    ReplyDelete
  161. @Scott

    Player -> Instrument -> Sound

    God -> ? -> Motion


    1. To what is the instrument analogous?


    2. Sound is not the creation of new music, nor does the sound's continued existence necessarily rely on the creation of new sound.

    ReplyDelete
  162. 2. Sound is not the creation of new music, nor does the sound's continued motion necessarily rely on the creation of new sound.

    Meant to write motion above.

    ReplyDelete
  163. (1) The analogue is Unmoved Mover -> Moved Mover -> Motion; in fact, our notion of 'instrument' comes from the notion of an instrumental cause (=moved mover). But your handling of the analogy is in any case unnecessarily clunky. If you don't want to talk about intermediaries, you can just as easily make the analogy

    [Player -> Instrument] -> Music
    [God] -> Motion

    Indeed, recognizing that you can do this is essential to understanding the First Way in the first place, since it's explicitly designed to recognize that principal causes and instrumental causes can function as a unity.


    (2) Nobody said it did; neither of these points is relevant to the analogy, and both of them engage in the equivocation Mr. Green noted about which change was the explanandum of a particular explaining cause, in this case by shifting what motion is being explained.

    ReplyDelete
  164. @Brandon

    [Player -> Instrument] -> Music
    [God] -> Motion


    Doesn't this actually cast the unmoved mover a moved mover?

    ReplyDelete
  165. Why would it? It's an analogy; your question is like asking whether it requires that all changes be vibrations in the air. The analogy was put forward to address a very specific objection you were making, to which instrumental causation doesn't seem particularly relevant, and to which you haven't shown that it is relevant; if, on the other hand, you are changing the objection itself, it is hardly surprising that TOF's analogy doesn't address an objection it was never formulated to address.

    ReplyDelete
  166. @Brandon

    I am confused because as far as I can tell, I am still objecting to the same issue to which I originally objected.

    This:

    it is necessarily the fact that, when the first mover is removed or ceases to move, no other mover will move or be moved.

    The side issue has become the relevance of an analogy which does not seem to resolve the objection without reading something into the analogy that happens to be exactly what I am objecting to in the first place.

    I will need to think about this...

    ReplyDelete
  167. @Bob

    A perfect analogy would no longer be a likeness, but the thing itself. Only the thing itself is like itself in *all* respects. One has to distinguish the relevant parts of an analogy from the irrelevant. Criticizing an analogy because its irrelevant parts are not like the original thing is a way of tuning out.

    But let’s see if we can find one you may find more congenial. Will an electric car continue to move when the power is cut?

    ReplyDelete
  168. @Paul

    Will an electric car continue to move when the power is cut?

    Newton says yes.

    ReplyDelete
  169. @Bob

    "Newton says yes."

    I don't know what you mean.


    ReplyDelete
  170. Bob, your scientism is showing. Inertia is not a problem. When the motor is cut, there is nothing propelling the car. Certainly, it may coast for a short time, but its motion has stopped, and the car has no power in itself to continue moving. The motor gave it momentum, and is no longer doing so. Eventually, it loses the momentum it has already been given by another (the motor) because the action of a second agent, friction, changes the prior motion.

    You are confusing the meta-physical issue with the physics of location change. See here for comments:
    https://docs.google.com/file/d/0B7SKlRTfkUieN3dGVkhNTi1SQUU/edit
    or
    http://faculty.fordham.edu/klima/SMLM/PSMLM10/PSMLM10.pdf
    +++++

    the simple math of the matter

    The math is not so simple; esp. if you front load it with all sorts of unspoken assumptions about probabilities, independence, and so on. More and more we find that genetic change can be "massive, sudden, and particular." For example, there are genetic mechanisms that attempt to incorporate glitches into the genome by reworking other segments.
    http://shapiro.bsd.uchicago.edu/Shapiro.2013.Rethinking_the_%28Im%29Possible_in_Evolution.html

    ReplyDelete
  171. @TheoFloinn

    Nothing to do with Scientism. The principle we are discussing simply seems not to be true in the most general sense and harms the overall metaphysical argument as a result.

    ReplyDelete
  172. @Paul Amrhein,

    "If the universe is like our deck of cards in being a mixture of act and potency lacking the power to realize its own potentials, then…what? Its potentials must be realized by something outside it."

    If the universe is a card, what does the remainder of the deck look like? This analogy clearly shows what's fundamentally wrong with the act/potency thing as applied to being itself. With cards, we simply reshuffle what's already there. Outside the universe, how does this apply? What does it mean for the universe to be reshuffled? -- reshuffled with what? How does a "potential" full house apply to nothing (a non-universe) becoming a full house? You might as well say a vacuum has the potential of becoming a full house. The goal of the reasoning is to somewhere assert "non-being" (a non-universe) has the "potency" to come into or go out of existence. That's simply not happening with the cards. They are in existence already. Rather than card reshuffling, the proper analogy would be a magician's card trick where he makes the deck disappear then reappear out of thin air. Then the supernatural element is made obvious from the beginning rather than sneaking it in at the end.

    I would be happy to discuss this elsewhere. Point me to a forum. Or email me at donjindra@hotmail.com

    @TheOFloinn,

    "The motor gave it momentum, and is no longer doing so."

    That's your human subjectivity showing. You select the motor to stand in for the "Prime Mover" or "First Cause" but why not the electric power station that moves the motor? Why not the coal that powers the station? Why not the miners who supply the coal? You selectively punch your way into the middle of an infinite chain and claim you've located the beginning of a "per se" chain. You've done no such thing. Such a chain exists nowhere in nature except as a concept in the human mind.

    ReplyDelete
  173. @Bob

    My time is short today. Please explain about the car not stopping. Or if you have already please point me to where.

    ReplyDelete
  174. Minor nitpick, but wouldn't the coal miners be per accidens? If they all suddenly die while the car is being propelled, the engine will continue on just fine.

    ReplyDelete
  175. @Anonymous,

    "but wouldn't the coal miners be per accidens? If they all suddenly die while the car is being propelled, the engine will continue on just fine."

    But is there anything but per accidens in the entire system? Let's look at that motor. It does not apply continuous force. If it's a brushless DC motor, for example, the electronic motor controller sends a series of pulses to coils to force the magnets to rotate to the next position. What we humans see as continuous rotation is due to the limitations of our senses. In reality that motor is pulsed. Rotation is not constant. Therefore the car's forward motion is driven by a per accidens series of pulses -- like a hammer hitting a nail repeatedly. Our sensation of a constant force at our backs is a fluke of biology. Our subjective sensation hides the fact that in objective reality this is no start of a per se chain. The same sort of analysis can be applied to music.

    ReplyDelete
  176. I haven't read up on motors, but isn't electricity required to sustain the electromagnet? No flow, no magnetic attraction/repulsion.

    Based on my understanding, per se chains are a result of something's lack of causal power if the "preceding" element does not actualize it's potential. The motor is not going to function in little bursts without having something actualize it in little bursts.

    ReplyDelete
  177. This comment has been removed by the author.

    ReplyDelete
  178. Let’s try this.

    Actually, the Newton thing only makes the point. Say our electric car is out in space. Cut the power, the car’s motion *in the relevant sense* stops. Without something to act on it, its trajectory will not change. For Aristotle and Aquinas, motion means change, one kind of change standing for change in general.

    Generally, when an agent ceases to act on an instrumental cause, the instrumental cause ceases to act *as an instrument*.

    ReplyDelete
  179. Or try this.

    If I said that when a living thing dies it stops moving (changing), would you object? I would say that the body stops changing as an instrument of the soul. Yes it goes on changing, but not in the relevant sense.

    ReplyDelete
  180. Anonymous,

    "No flow, no magnetic attraction/repulsion."

    Right, however it's caused by the flow of not one but billions of charged particles (electrons). Which one of those billions do we single out and say, Here starts the per se chain? I think you can see my problem with that. It is an interesting example though. I need to give it some thought. My first thought is that I doubt it's a good example for applying to the cosmological argument because that argument is in search of one prime mover, not the billions required to support a decent magnetic field.

    "Based on my understanding, per se chains are a result of something's lack of causal power if the 'preceding' element does not actualize its potential."

    I don't think that's quite right. Feser puts it this way: "[I]t is because all the causes in ... a series other than the first are instrumental ... that they are said to be ordered per se or 'essentially,' for their being causes at all depends essentially on the activity of that which uses them as instruments."

    Instrumentality is the key.

    ReplyDelete
  181. Which one of those billions do we single out and say, Here starts the per se chain?

    Why should any one of them do so?

    ReplyDelete
  182. The magnetic field example got me to thinking. Surprisingly it led me to a better understanding of why I've never thought the per se versus per accidens dichotomy was legitimate.

    When thinking about electricity it's good to conceptualize it as water. This got me to a toy boat in a bathtub. A kid learns he can move the boat by shoving water at it. He never has to touch the boat. But he could take a stick and push it through the water too. It's obvious to me that when using water alone to move the boat, there is no per se chain. The hand pushes the water, and the current and/or waves radiate from the hand and eventually move the boat. But the water does have a "power" of its own. It will slosh against the side of the tub quite apart from the intention of the kid or his subsequent hand movement. The waves of water keep the boat bobbing. Each molecule of water moves independently. The hand may have started the body of water's motion, but once affected, the molecules of water are then free to cause other events quite separate from the hand's presence. So they are not being used instrumentally by the hand.

    This led me to frozen water on a mountain. A skier starts an avalanche. The snow and ice tumble downhill and destroy a cabin. It's not a per se chain that buries the cabin. It's the cumulative effect of billions of interconnected events.

    Magnetic fields are like this. It's the cumulative effect of billions of spinning electrons. Each electron causes, and always causes, an electromagnetic disturbance. The fields that move magnets in a motor are the cumulative effect of billions of electrons moving in a more orderly per accidens chain.

    Finally, this is exactly what the hand-stick-stone analogy is too. It's the hardness of the stick to our senses that confuses the issue. In reality, that stick is just like that water in the tub. The molecules move in a more uniform manner but they do slosh at the atomic level. It cannot then be per se.

    Paul's instrumental movement of the body by the "soul" remains the most interesting example.

    ReplyDelete
  183. "The actual situation, then, is this. Examples like the hand pushing the stone etc. have (like the physicist’s examples of riding a beam of light or falling into a black hole) no significance other than as loose illustrations of certain abstract concepts -- in this case the concepts of instrumental causality, the actualization of potency, and the like. The actual physical details are completely irrelevant, just as the fact that you’d be torn apart if you fell into a black hole and the fact that a photon is too small to sit on are completely irrelevant to the points the physicist is trying to make. And just as it would be silly to harp on the impossibility of riding on a photon or surviving a fall into a black hole as proof that a certain physicist hasn’t gotten his physics straight or that he doesn’t care about the actual empirical facts, so too is it silly to harp on the physics of the local motion of sticks and stones as proof that Aquinas, Feser, et al. haven’t gotten our physics straight or that we don’t care about the actual empirical facts."

    http://edwardfeser.blogspot.com/2012/11/the-incompetent-hack.html

    ReplyDelete
  184. Anonymous,

    Thanks for the link. I missed that one when originally posted. As a brief illustration of what I think of the reasoning there, suppose I said this:

    Examples like Anne Rice's Interview with the Vampire have no significance other than as loose illustrations of certain abstract characteristics of vampires Nevertheless, those 'loose illustrations' are evidence enough to take vampires seriously. The actual physical fact that there are no vampires is completely irrelevant.

    The monster with no reflection in the mirror, in this case, is instrumental cause. Any plot that derives from it, no matter how elaborate and entertaining, is just more fiction.

    (For someone who so quickly spots begging the question in others, it would be nice if Feser used that talent to examine Aquinas.)

    ReplyDelete
  185. If the things we talk about, cars, tables, are illusory, when can we speak the truth? Can “The cup I have just put on the table is no longer moving.” be true in some way if at the subatomic level it is false? Can a statement be true at one level and false on another?

    What should we make of Einstein’s thought experiment where he imagined riding on a beam of light? Can an impossibility not illustrate a truth?

    ReplyDelete
  186. Don Jindra,

    Your reasoning doesn't parallel Feser's answer to the objection. For example, where do Feser/Aquinas imply this "Nevertheless, those 'loose illustrations' are evidence enough to take vampires seriously." (or its parallel)? Feser/Aquinas do not use illustrations as evidence.

    This is also from the link:

    "As Rudi te Velde has suggested, some critics place too much significance on the physical details of the examples Aquinas gives in the course of the proof, failing to see that their point is merely to illustrate certain basic metaphysical principles rather than to support broad empirical or quasi-scientific generalizations. (p. 68, emphasis added)"

    ReplyDelete
  187. “But the water does have a "power" of its own. It will slosh against the side of the tub quite apart from the intention of the kid or his subsequent hand movement.” DJ

    How does the water have a power of its own? Wouldn’t it keep radiating outward *on its own*?

    ReplyDelete
  188. “The hand may have started the body of water's motion, but once affected, the molecules of water are then free to cause other events quite separate from the hand's presence. So they are not being used instrumentally by the hand.” DJ

    But such counter-waves can be used intentionally and instrumentally. This is often done, for example, in the placement of loudspeakers or microphones. I think you would have to show that such counter-waves *cannot* be used instrumentally for the point to go through.

    ReplyDelete
  189. Anonymous,

    "Feser/Aquinas do not use illustrations as evidence."

    The distinction between between per se and per accidens is based wholly on illustrations. Where else would it come from? Logic? No.

    "As Rudi te Velde has suggested, some critics place too much significance on the physical details of the examples Aquinas gives in the course of the proof, failing to see that their point is merely to illustrate certain basic metaphysical principles rather than to support broad empirical or quasi-scientific generalizations. (p. 68, emphasis added)"

    What else could he say when the illustrations don't truly illustrate the concepts? It's not too much to ask for an objective example where the physical details do precisely what the concepts claim they do. Otherwise we have no reason to believe the concepts are based on physical reality. Let's remember, the goal is to better describe being itself. You cannot do that by using concepts that have no objective basis in physical reality.

    Paul,

    I'll answer you privately.

    ReplyDelete
  190. "The distinction between between per se and per accidens is based wholly on illustrations. Where else would it come from? Logic? No."

    Philosophy of nature/metaphysics of change. It's in the link.

    "What else could he say when the illustrations don't truly illustrate the concepts? It's not too much to ask for an objective example where the physical details do precisely what the concepts claim they do. Otherwise we have no reason to believe the concepts are based on physical reality. Let's remember, the goal is to better describe being itself. You cannot do that by using concepts that have no objective basis in physical reality."

    Physics requires a high level of abstraction. You have to watch out for this before claiming there really is no per se in nature:

    http://edwardfeser.blogspot.com/2012/08/concretizing-abstract.html

    ReplyDelete
  191. Anonymous,

    "Philosophy of nature/metaphysics of change. It's in the link."

    The link is mainly a defense against Hallquist's charges. It doesn't develop the metaphysics. That's done elsewhere and I know it pretty well by now.

    Regarding "Concretizing the abstract," the text can't justify abstracting an error then using it as truth.

    ReplyDelete
  192. "The link is mainly a defense against Hallquist's charges. It doesn't develop the metaphysics. That's done elsewhere and I know it pretty well by now."

    But it does point toward where the concepts come from, which was your question.

    "Regarding "Concretizing the abstract," the text can't justify abstracting an error then using it as truth."

    That isn't why I posted the link.

    ReplyDelete
  193. Case 1 slow-motion

    (The hand moves the stick.) The stick hits the rock (action). The rock starts moving and resisting (reaction). The motion of the stick slows down a little or a lot depending on the size of the rock (resultant). Let’s say that the hand pushes the rock an inch northward then stops. Has the rock caused itself to move in precisely that way? I say no. What was the cause of that motion? I say it was principally the hand. The accent in this example is on instrumentality. Something is needed to move the rock and the stick, something of a different order from either of them.

    Case 2 stick breaks

    (The hand moves the stick.) The stick hits the rock (action). The heavy heavy rock starts moving slightly (with the hand) and resisting mightily (reaction). The opposed forces on the stick exceed its compression strength. It breaks (resultant). What caused the stick to break? Was it the action of the hand (plus stick) alone? I say no. Was it the reaction of the rock alone? I say no. Was it the two forces acting together, that is, simultaneously? I don’t know what other conclusion is possible. The accent in this example is on the simultaneity of cause and effect. If either force were to stop acting, the stick wouldn’t continue to break. Here action, reaction, and resultant are simultaneous, or at least overlapping significantly (in time).

    ReplyDelete
  194. Well, it's not a general theory of sticks and stones, only an illustrative example of an instrumental mover. In a different context, you may illustrate a different point. It still comes down to this: that an instrument cannot play itself. The stick has no power to move a stone unless it is being moved by another. Once the hand stops moving the stick, the stick stops moving the stone, because the stick cannot act on its own. This remains true even if the rock continues to roll (or to rock around the clock) due to Bradwardine's "first and last moments." The stick isn't moving it any more.

    ReplyDelete
  195. Ed uses the term "simultaneous" to describe the other kind of causality than temporal. I find this an unfortunate word, because this still makes it sound somehow related to time. Modern people tend to think of causality in exclusively temporal terms, and the word "simultaneous" does not clarify properly that what is meant in this case is the kind of causality perpendicular to temporal causality.

    "Instrumentality" is a bit better term. For me, "hierarchical causality" sounds better still, and clearer.

    ReplyDelete
  196. @E. Seigner

    I like the term “vertical” causality. I also like the phrase “relational holism.” All of this reminds me of a Sufi parable that runs more or less as follows.

    A- Why were you chasing me?
    B- Because you were running. Why were you running?
    A- Because you were chasing me.

    ReplyDelete